Sie sind auf Seite 1von 263

A SEQUENCE OF SEVERAL TERMS IS CALLED A SERIES.

EACH UNIT OF A SEQUENCE OR SERIES IS CALLED A TERM OR AN


ELEMENT.
ALL TERMS IN A SERIES FOLLOW A CERTAIN PATTERN THROUGH
OUT THE SEQUENCE.
EACH TERM IN THE SERIES IS IMPORTANT BECAUSE THERE EXISTS
A CERTAIN RELATIONSHIP BETWEEN TWO CONSECUTIVE TERMS.
THERE ARE TWO TYPES OF SERIES.
I. ENGLISH ALPHABET SERIES AND
II.MIXED SERIES.

ENGLISH ALPHABET SERIES


In this type of questions, a series of single, pairs or
groups of letters or combination of letters and numerals
is given.
The terms of the series form a certain pattern as regards
their position of letters in the English Alphabet.
The candidate is required to decipher this pattern and
accordingly find out the required answer.

ENGLISH ALPHABET SERIES.

Eg. A B C D E F G H I J K L M N O P Q R S T U V W X Y Z
Qn. i). Find the eighteenth letter in the above alphabet series.
Ans. The 18th letter counting from the left end is R.
Qn. ii). Find the tenth letter from the right.
Ans. When we count from the right to the left, the tenth letter is Q
This is an example of a simple series.
Questions on this are relatively simple.
Qn. iii) Find the next two terms in the series : A, C, F, J, ? , ?
(a) L,P (b) M,O (c) O,U (d) R,V (e) K, L
Solution:
It is clear that the first, second, third..letters of the series are
respectively moved two, three, four ..steps forward to obtain the
successive terms in the series.
Thus, the fifth term in the series must be a letter five steps ahead of J
ie., O and the sixth term in the series must be a letter six steps ahead of
O ie., U.
So, the pattern is A +2 C+3
F +4
J +5
O +6
U

Hence, the missing terms are O and U and


the answer is (c).

ALPHABET SERIES.
Alphabet Series consists of letters of the
alphabet placed in a specific pattern.
If you keep in your mind the order of the
letters with their respective numbers, it will
help you answer the question quickly.
1 2 3 4 5 6 7 8 9 10 11 12 13
A B C D E F G H I J K L M
26 25 24 23 22 21 20 19 18 17 16 15 14
14 15 16 17 18 19 20 21 22 23 24 25 26
N O P Q R S T U V W X Y Z
13 12 11 10 9 8 7 6 5 4 3 2 1

ENGLISH ALPHABET SERIES.

Illustrative Example
What will be the next term in:
BKS,DJT,FIU,HHV,?
(a) IJX (b) IGX (c) JGW (d) IGU (e) JGU
Solution:
Ans: (c) JGW
Explanation: In each term, the first letter is
moved two steps forward, the second letter
one step backward and the third letter one
step forward to obtain the corresponding
letter in the next term.
So, the missing term is JGW.

LETTER SERIES
This type of questions usually consists of a series of small letters
which follow a certain pattern.
Among these, some letters are missing in the series.
The missing letters are given in one of the five answer options which
you have to choose from the alternatives.

Illustrative Example
aab aaa bba (a) baa (b) abb (c) bab (d) aab (e) bbb
Solution:
Ans: (a) baa
Explanation: The first blank space should be filled in by b so that we
have two as followed by two bs.
The second blank space should be filled in either by a so that we have
four as followed by two bs or by b so that we have three as
followed by three bs.
The last blank space should be filled in by a.
Thus, we have two possible answers baa and bba.
But only baa appear in the alternatives.
So, the alternative (a) baa is the answer.

LETTER SERIES.

Illustrative Example

Directions- (Q. 01-05): In the following questions, some of the letters are
missing. The missing letters are given in one of the proper sequence
as one of the alternatives given under each question. Find out the
correct choice.
01. ab- abb-bba-b
(a) aba (b) bba (c) bab (d) baa (e) None of these
Solution: Ans: (c) bab
Explanation: The series formed is: abb / abb / abb / abb / abb
The missing letters are bab. So, the option, (c) bab is the right answer.
02. rst- - rs-uv-stu-rst- (a) rstrsts (b) uvtrstu (c) uvtrstv (d) uvtrvuv (e) uvtrsuv
Ans: (d) uvtrvuv
Explanation: The series formed is: rstuv / rstuv / rstuv / rstuv / rstuv
The missing letters are uvtrvuv. So, the option, (d) is the right answer.
03. c-ca-ab-bc- (a) abcac (b) babca (c) ccabb(d) bcabb (e) bbcbb
Ans: (b) babca
Explanation: The series formed is: bca / cab / abc / bca / cab /
The missing letters are babca. So, the option, (b) is the right answer.

LETTER SERIES.

Illustrative Example

Directions- (Q. 04-05): In the following questions, some of the


letters are missing. The missing letters are given in one of
the proper sequence as one of the alternatives given under
each question. Find out the correct choice.
04. ab -b- bc-ca(a) cacab (b) abca (c) abacb (d) accbb (e) None of these
Solution: Ans: (a) cacab
Explanation: The series formed is: cab / abc / bca / cab / abc.

The missing letters are cacab. So, the option, (a) bab is the
right answer.
05. a- -ba-b-aa-b
(a) aabab (b) ababb (c) bbaba (d) baaba (e) abcbc
Ans: (b) ababb
Explanation: The series formed is: aabb / aabb / aabb / aabb /
The missing letters are ababb. So, the option, (b) is the
right answer.

CORRESPONDENCE SERIES

This type of series questions usually consists of a three sequences with three
different elements (usually Capital letters, digits and small letters ). On the
basis of similarity in positions in the three sequences, a capital letter is found to
correspond with a unique digit and a unique small letter, whenever it occurs.
The candidate is required to trace out this correspondence and accordingly
choose the elements to be filled in at the desired places.
Illustrative Example
In the following series, choose the alternative which contains the numerals to be
filled in the marked places, in the correct order.
B - - D - - CAB DA C B
- -4 132- - - ? ? ? ?
aabcc- - - - (a) 1,2,3,4 (b) 2,3,1,4 (c) 1,2,4,3 (d) 2,1,4,3 (e) 4,1,2,3
Solution: Ans: (a) 1,2,3,4
Explanation: In the second series 1 corresponds to D in the first series. So
question mark below D is to be replaced by 1. In the third series, c at the 8th
place corresponds to A in the first series while c at the sixth place
corresponds to 2 in the second series. So 2 corresponds to A. So, A is to be
replaced by 2. In the third series, a at the first place corresponds to B in the
first series while a at the 3rd place corresponds to 4 in the second series. So 4
corresponds to B and to be replaced by 4. Now only 3 remains and it
corresponds to C and to be replaced by 3.
Thus, we have DACB corresponds to 1,2,3,4.

CORRESPONDENCE SERIES

Illustrative Example-1

Directions (Qns.01-05): In each of the following questions three


sequences with three different elements Capital letters, digits and
small letters are given which corresponds to each other in some way.
In each question, you have to find out the letters/ numerals that come
in the vacant places marked by (?). These are given as one of the four
alternatives under the question. Mark the answer as instructed.
CB--D- BABC C B
- -1243- - ? ? ? ?
aabc b - - - (a) 3,4,4, 3 (b) 3,2, 2,3 (c) 3,1,1,3 (d) 1,4,4,1 (e) 4,1,2,3
Solution: Ans: (c) 3,1,1, 3
Explanation: In the third series a in the first place corresponds to C in
the first series and a in the third position corresponds to 1 in the
second series. So question mark below Cs are to be replaced by 1s.
In the third series, b at the 4th place corresponds to 2 in the second
series while b at the 8th place corresponds to A in the first series. So
2 corresponds to A.
In the second series, 4 at the 5th place corresponds to D in the first series

Now only 3 remains and it corresponds to B and to be replaced by 3.


Thus, we have BCCB which corresponds to 3,1,1, 3.

CORRESPONDENCE SERIES

Illustrative Example-2

Directions (Qns.02-05): In each of the following questions three sequences


with three different elements Capital letters, digits and small letters
are given which corresponds to each other in some way. In each
question, you have to find out the letters/ numerals that come in the
vacant places marked by (?). These are given as one of the four
alternatives under the question. Mark the answer as instructed.
- AC-BD- C DC D
2 - 4 1- 1 4- - - cd - bca? ? ? ?
(a) a,b,a, b (b) a,c, a,c (c) c,b,c,b (d) c,d,c,d (e) b,a,b,a
Solution:
Ans: (a) a,b,a, b
Explanation: In the second series 4 in the 3rd place corresponds to C in
the first series and a in the 7th position corresponds to 4 in the second
series. So question mark below Cs are to be replaced by as.
Similarly 1 and 2 corresponds to D and B in the first series and D
and c and b in the third series.
Thus, we have CDCD which corresponds to 4,1,4,1 in the second and
a,b,a, b in the third series.

So, the positions of (?)s are to be filled with option a) a,b,a, b

CORRESPONDENCE SERIES

Illustrative Example-3

Directions (Qns.03-05): In each of the following questions three


sequences with three different elements Capital letters, digits and
small letters are given which corresponds to each other in some way.
In each question, you have to find out the letters/ numerals that come
in the vacant places marked by (?). These are given as one of the four
alternatives under the question. Mark the answer as instructed.
C -B -D - A - B BDD
2 - - 4 - 3 4 - ? ? ? ?
- a - c ba d - - - (a) 2,2,1, 1 (b) 2,2,3,3 (c) 3,3,4,4 (d) 3,3,1,1 (e) 3,3,2,2
Solution: Ans: (d) 3,3,1,1
Explanation: In the second series 2 in the 1st place corresponds to C
and 4 corresponds to A in the first series.
Similarly 1 and 3 corresponds to B and D in the first series .
Thus, we have BBDD which corresponds to 3,3,1,1 in the second series.

So, the positions of (?)s are to be filled with option (d) 3,3,1,1
Note:- There are two answer possibilities; 3311 and 1133.
1133 is not given in the alternatives. Hence, A=4, B=3, C=2, D=1.
So option (d) is accepted as the best choice as answer.

CORRESPONDENCE SERIES

Illustrative Example-4

Directions (Qns.04-05): In each of the following questions three


sequences with three different elements Capital letters, digits and
small letters are given which corresponds to each other in some way.
In each question, you have to find out the letters/ numerals that come
in the vacant places marked by (?). These are given as one of the four
alternatives under the question. Mark the answer as instructed.
A - BAC - D - B CDC
- 3 - 2 - 1 - 4? ? ? ?
d c - - ba c b - - - (a) 1,3,4,3 (b) 1,4,3,4 (c) 2,3,4,3 (d) 3,4,1,4 (e) 3,3,2,2
Solution:
Ans: (b) 1,4,3,4
Explanation: b in the 5th place corresponds to C and 4 corresponds to
b in the 3rd series.
Similarly c corresponds to D and 3 corresponds to c So, 3
corresponds to D.
Now only 1 remains and it corresponds to B.
Thus, we have BCDC which corresponds to 1,4, 3,4 in the second series.

So, the positions of (?)s are to be filled with option (b) 1,4,3,4

CORRESPONDENCE SERIES

Illustrative Example-5

Directions (Qns.05-05): In each of the following questions three


sequences with three different elements Capital letters, digits and
small letters are given which corresponds to each other in some way.
In each question, you have to find out the letters/ numerals that come
in the vacant places marked by (?). These are given as one of the four
alternatives under the question. Mark the answer as instructed.
- ADACB - - B DC C
1 3 - - 1 2 4 2- - - a - - b -- - c d ? ? ? ?
(a) a,c,d,d (b) a,d, c,c (c) c,a,d,d (d) d,c,a,a (e) b,a,b,a
Solution:
Ans: (d) d,c,a, a
Explanation: b in the 4th place corresponds to A
1 corresponds to C and a corresponds to 1. So, a corresponds to C.
2 corresponds to B and d corresponds to 2. So, d corresponds to
B.
Now remaining letter is only c and it corresponds to D.
Thus, we have BDCC which corresponds to d,c,a, a in the 3rd series.

So, the positions of (?)s are to be filled with option (d) d,c,a, a

NUMBER SERIES
Case-1: Completing the given series by finding the missing
term (s)
Directions: Find the missing term in each of the given series,
Eg. 2, 5, 9, 19, 37, ?
(a) 73 (b) 75 (c) 76 (d) 78 (e) 79
Solution:
It is clear that the given sequence follows a pattern :
2 x 2+1 =5, 5x2-1=9, 9x 2 +1=19, 19 x 2-1 = 37.and so on in the
successive terms in the series.
So, the missing term is 37 x2 + 1 = 75
Hence, the answer is (b)

i. Arithmetic Progression (AP):


The progression in the form a, a + d, a + 2d, a + 3d, a + 4d is
known an A.P. with first term is = a and common difference is
= d.
Example: 3, 6, 9, 12, . is an AP with a = 3 and d = 6 3.
In an AP we have nth term = a + (n-1)d.

ILLUSTRATIVE QUESTION
Qn. iii) In the series 3, 9, 15,.what will
be the 21st term?
(a)117P (b) 121 (c) 123 (d) 129 (e) 131
Solution:
It is clear that the 3 +6 = 9, 9 + 6 = 15, so the
series is an AP in which a =3 and d = 6.
Therefore, the twenty first term in the series
must be a + (21-1) d
= a+ 20d = 3 + 20 x 6 = 3 + 120 = 123
So, the 21st term is 123 .
Hence, the answer is (c).

ii. Geometric Progression (G.P.)

The progression of the form a, ar, ar2,ar3,is known as a G.P.


with first term = a and common ratio = r.
Ex. 1,5,25,125,.. Is a G.P. with a = 1 and r = 5/1= 25/5 =..= 5

In a G.P., we have nth term = arn-1


Qn. In the series 7, 14, 28, .. what will be the tenth term?
(a)1792 (b) 2456 (c) 3584 (d) 4096 (e) 4196
Solution:
The given series is a GP in which a =7 and r =2
Therefore 10th term = nth term = arn-1 = ar10-1
= 7x 29 = 7 x 512 = 3584
Hence, the answer is c

NUMBER SERIES
Case-2: Finding the wrong term in the given series
Directions: In each of the following questions, one term in the number
series is wrong. Find out the wrong term.
Eg.i. Find out the wrong number in the series.
7, 28, 63, 124, 215, 342, 511
(a) 7 (b) 28 (c) 124 (d) 342 (e) 511
Solution:
It is clear that the correct sequence is :
23 - 1, 33 -1, 43-1, 53-1, 63-1, 73-1, 83-1.and so on in the successive terms
in the series.
So, 28 is the wrong term and should be replaced by 33-1 = 26
Hence, the answer is (b)

Ex.2. Find out the wrong number in the series.


3,8,15,24,34,48,63
(a) 15 (b) 24 (c) 34 (d) 48 (e) 63
The difference between consecutive terms of the given series are
5,7,9,11,13 and 15 respectively.
So,34 is the wrong term and should be replaced by (24 + 11) = 35

Hence, the answer is (c)

IT CONSISTS OF LETTERS NUMBERS AND SYMBOLS


THE NUMBER IN THE MIXED SERIES IS NOT FIXED
IT CAN HAVE ANY NUMBER OF ELEMENTS LIKE 24, 28 OR
MORE.
QUESTIONS ON SERIES JUDGE HOW QUICKLY ONE CAN
IDENTIFY THE RELATIONSHIP BETWEEN THE GIVEN
ELEMENTS.
QUESTIONS ON SERIES CAN BE CLASSIFED IN TO TWO
TYPES.
I. SEREIS CONSISTING ON LETTERS AND NUMBERS

Eg. TGL8AVER6MI3 And


II. SEREIS CONSISTING ON LETTERS , NUMBERS AND
SYMBOLS

Eg. =NR<AF3@LMZP

I. SEREIS CONSISTING ON LETTERS AND NUMBERS


Eg. TGL8AVERM6I3UZDSW75FJBYQ21K
Find out which letter/number is forth to the left of the sixteenth letter
from your left.
Find the sixteenth letter /number from the left which in this case, is S.
Then fourth letter to the left of the 16th letter /number to the left of S is
3
In short you are asked to find out 16-4 = 12th number from left.
II. SEREIS CONSISTING ON LETTERS , NUMBERS AND SYMBOLS
Eg., Study the following II. series of letters , numbers and symbols.
= N R <2 A F 3 @ L M Z P > Q 5 4 R T J 6 K L # $ 9 8 * 3
I. If the first thirteen elements are written in the reverse order, which
letters / numbers/ symbol will be the sixth to the left of the 12 th letters /
numbers / symbols from your right?
Solution: When the thirteen elements are written in the reverse order,
the swequence will be as follows:P Z M L @ 3 F A2 <R N = > Q 5 4 R T J 6 K L # $ 9 8 * 3
The twelfth letter from the left is N and the sixth element to the left of
N is 3.

REMEMBER
THE FOLLOWING IMPORTANT POINTS
GIVEN BELOW WHEN SOLVING
QUESTIONS ON SERIES.
1. QUICKLY COUNT THE TOTAL NUMBER OF
TERMS IN THE GIVEN SERIES.
2.LOCATE THE MIDDLE TERM IF ANY.
WRITE DOWN THE TOTAL NUMBER OF
LETTERS , NUMBERS AND SYMBOLS
RESPECTIVELY

ILLUSTRATIVE QUESTION-1

Directions:- (Q.Nos.1 -5):


Study the following letter-number symbol series and
answer the questions follow:
KD4*NTZ#<6P@5MA2$1VCR>9E =
1. If the first five elements and last ten elements are written
in the reverse order, which of the following will be the
seventh element to the right of eleventh element from
your left.
(1) =
(2) 2 (3) 9 (4) E (5) None of these
Ans. (3) 9
When the first five elements and last ten elements are written
in the reverse order, the sequence would be as shown
below.
N*4DKT Z #<6P@5MA = E9 >RC V1 $ 2
Then, eleventh element from your left is P and the seventh
element to the right of P is 9.
Try the exercise without writing in the reverse order, instead
the required number of letters in the reverse order.

ILLUSTRATIVE QUESTION-2
Which of the following would be exactly midway between the ninth element
from the left and the eleventh element from your right?

KD4*NTZ#<6P@5MA2$1VCR>9E =

(1) M (2) P (3) 5 (4) @ (5) None of these

Ans. (4) @
The ninth element from your left is < and the eleventh element from the right is A.

So, the element midway between < and A is @


3. Four of the following five are alike with respect to their positions in the
above series and hence, form a group. Which one does not belong to
the group?
(1) DT< (2) P2C (3) 5 $ C (4) T 6 5 (5) @ 2 V
Ans. (2) P2C
i.e.,

<

As can be seen, P2C does not belong to the group

ILLUSTRATIVE QUESTION- 4. What should come in place

of Question Mark (?) in the following series on the


basis of above sequence?
KD4*NTZ#<6P@5MA2$1VCR>9E =
N#A, Z@$, <2V, PCR, ?
(1) M E 5 (2) 5 E 9 (3) @ = 5 (4) 2 > 9 (5) None of these

Ans. (2) 5E9


Each element in the group is connected to the
corresponding element in the next group as follows.

N
A
Z
$
<
P
V
R

Z = +2
$ = +2
< =+2
V =+2
P=+ 2
5=+ 2
R=+2
9=+2

Similarly the middle elements


are
#
@ = +4
@
2 = +4
2
C = +4
C
E = +4

Therefore, the place at (?) should be 5E9

ILLUSTRATIVE QUESTION- 5. Which of the following would


be 4th to the right of M, if all the elements after M are written
in the reverse order?

KD4*NTZ#<6P@5MA2$1VCR>9E =
(1) =

(2) E

(3) > (4) V (5) None of these

Ans. (3) >


When the elements after M is written in the reverse
order, the sequence is as follows:
KD4*NTZ#<6P@5M =E9>RCV1#2A
THE FOURTH ELEMENT TO THE RIGHT OF M IS >.

Short Cut: We are asked to find the fourth element


to the right of M after the remaining letters are
reversed.
We need not write the new series but count the 4th
letter from the right, which is >

ILLUSTRATIVE QUESTION-6
Directions: (Qn.6-10): Study the following
arrangement of numbers, letters and
symbols carefully and answer the questions
given below:B@KI9D52T$M4EJ#UR1AK5W6%FQ7HP8Z
06. How many such symbols are there in the
above arrangement each of which is
immediately preceded by a letter but not
immediately followed by a letter?
(1) None (2) One (3) Two (4) Three (5) more than 3

Ans: (2) One


There is only one such symbol

ILLUSTRATIVE QUESTION-7
Directions: (Qn.7-10): Study the following
arrangement of numbers, letters and
symbols carefully and answer the questions
given below:B@KI9D52T$M4EJ#UR1AK5W6%FQ7HP8Z
07. Which of the following is ninth to the right
of the twentieth element from the right end
of the above arrangement?
(1) 9 (2) 7 (3) $ (4) 5 (5) None of these

Ans: (4) 5
Note:- Nineth to the right of 20th element from the
right means 20 9 = 11th element from the right
which is 5.

ILLUSTRATIVE QUESTION-8
Directions: (Qn.8-10): Study the following
arrangement of numbers, letters and symbols
carefully and answer the questions given
below:B@KI9D52T$M4EJ#UR1AK5W6%FQ7HP8Z
08. Which of the following should come in
place of (?) in the following series based on
the elements in the above arrangement?
K9D, 2$M , JUR, ?
(1) KW (2) K5 (3) 56 (4) W6 (5) None of these
Ans: (5) None of these
+2 +1+3 pattern gives 5W which is not an
alternative given.

ILLUSTRATIVE QUESTION-9
Directions: (Qn.9-10): Study the following
arrangement of numbers, letters and
symbols carefully and answer the questions
given below:B@KI9D52T$M4EJ#UR1AK5W6%FQ7HP8Z
09. How many such consonants are there in
the above arrangement each of which is
immediately preceded by a number and
immediately followed by a symbol?
(1) None (2) One (3) Two (4) Three (5) more than 3

Ans: (3) Two


There are two such consonants, D and T

ILLUSTRATIVE QUESTION-10
Directions: (Qn.10-10): Study the following
arrangement of numbers, letters and
symbols carefully and answer the questions
given below:B@KI9D52T$M4EJ#UR1AK5W6%FQ7HP8Z
10. How many such numbers are there in the
above arrangement each of which is
immediately followed by a symbol but not
immediately preceded by a consonant?
(1) None (2) One (3) Two (4) Three (5) more than 3

Ans: (1) None


There are no such numbers.

ILLUSTRATIVE QUESTION-3
Directions:- Find the missing term in the
following series.
1,6,15, ?, 45, 66, 91
(a) 25 (b) 26 (c) 27 (d) 28 (e) 29
Solution: The sequence follows a pattern
+5, +9, +13, + 17, +21, +25
Thus 1+5 = 6, 6+9=15, 15+13 = 28.
So, missing term = 15+13 = 28
Hence the answer is (d)

ILLUSTRATIVE QUESTION-6
Directions:- In each of the following questions,
one term of the number series is wrong. Find out
the wrong term.
69, 55, 26, 13, 5
(a) 5 (b) 13 (c) 26 (d) 55 (e) 29
Solution: In the given series, each term is 1 more
than the product of the digits of the preceding
Thus (6 x 9)+1 = 55, (5x5)+1=26, (2x6)+1 = 13.

So, 5 is wrong and must be replaced by


(1x3)+1 = 4
Hence the answer is (a)

ILLUSTRATIVE QUESTION-7

Directions:- In each of the following number series two


terms have been put within brackets. Mark your answer as
(a) If both bracketed terms are right
(b) If the first bracketed term is right and second is wrong
(c) If the first bracketed term is wrong and second is right
(d) If both the bracketed terms are wrong
(e) None of these
4, 6, 10, (12), 16, (14), 22
Solution: The correct pattern is +2, +4, +2, +4,.
Clearly, the term 12 is correct.
But, 14 is wrong and must be replaced by (16+2) ie., = 18

Hence the answer is (b)

Coding is a method of transmitting a message from the


sender to the receiver in a code language which cannot
be understood or comprehended by the third person.
The coded message can be deciphered or decoded by
the receiver as he knows the rule of decoding the
message.
Coding decoding test is given to judge the candidates
ability to decipher the rule which is applied for coding a
particular message and break the code to reveal the
message.
In these type of questions, some words or group of
letters and their coded forms are provided and you are
required to find the rule of coding so that you can either
encode or decode another word or another group of
letters in a similar way.

CODE BASED ON ENGLISH ALPHABET

A
Z

In order to solve questions of this type, all you needed to know is the
position of each letter from either end so that you can recognize the
coding pattern quickly.
1 2 3 4 5 6 7 8 9 10 11 12 13
A B C D E F G H I J K L M
These matrices
are useful to find
26 25 24 23 22 21 20 19 18 17 16 15 14
the coding
14 15 16 17 18 19 20 21 22 23 24 25 26
pattern of letters
N O P Q R S T U V W X Y Z
quickly.
13 12 11 10 9 8 7 6 5 4 3 2 1
Also, one must know the series of opposite alphabets

B
Y

C
X

D
W

E
V

F
U

G
T

H
S

I
R

J
Q

K
P

L
O

M
N

(1). Code based on jumbled letters of the same word.


In this type the letters of the given word are jumbled
according to certain rules to create a new coded word.
(2) Code consisting of preceding/ succeeding letter: In these type of questions, either preceding or
succeeding (following) letter stand as the code letter for
each of the letters of the given word.
Sometimes, a combination of both preceding or following
letters are used to generate the code.
Example:1. If in a certain code, LUTE is written as MUTE
and FATE is written as GATE, then how will BLUE be
written in that code?
(a) CLUE (b) GLUE (c) FLUE (d) SLUE
Ans. 1. (a) CLUE. First letter of the code word is one step
behind the corresponding letter of the word.

3.Code consisting of a group of words.


In these type of questions, a whole sentence or a group of
word is coded.
By comparing any two given sentences and its code, you
can find the code word for each word on the basis of the
common word.
Example:1. In a certain code LIM SUK TA means boys are
clever, DIN LIM PU means girls are tall and PU NE TA
means Boys and girls. which word in the language means
and
(1)LIM (2) TA (3) DIN (4) PU (5) None of these
2. If (i) A*B means A is the brother of B (ii) A0B means A is
the daughter of B (iii) A+B means A is the sister of B
Which of the following show Pis the maternal uncle
of Q ?
(1) P+R0Q (2) P*R0Q (3) QoR+P
(4) QRP (5) None of these

4. Code consisting of numbers

In these type of questions, a group of words or a whole sentence is


coded as numbers and on the basis of common words or codes, one
can find the code word for a certain word.
Eg.i. CLOCK is 42145, LEAN is 2068, CARE is 4690, then NECKLACE is
(1) 80546240 (2) 6054842 (3) 80452640 (4) 50842604 (5) 80546204
ii. If ROSE is coded as 6821, CHAIR as 73456 and PREACH is coded as
961473, what will be the code for SEARCH?
(1) 246173 (2) 214673 (3) 214763 (4) 216473 (5) 216471
R
6, O
8,S
2, E
1, C
7, H
3, A
4, I 5, P 9
Hence, SEARCH = 214673

5. Code of meaningful Words

In these type of questions, certain objects or things are assigned


different names and you are asked to find the code word for a given
word in the code language.
Eg. If sky is called sea, sea is called water, water is called air, air is called
cloud and cloud is called river, then what do we drink when thirsty?
(1) sky (2) air (3) water (4) sea (5) water

6. Code of symbols
In these type of questions, certain symbols and
signs are used for coding words or letters.
7. Code involving mathematical operations with
the position number of letters in alphabet series.
In these type of questions, the words are coded by
certain numbers.
This number is obtained by performing an
arithmetical operation of the number of each letter
in alphabet series.
Eg. 1,2,6, 24 (------)

(1) 60 (2) 120 (3) 95 (4) 150 (5) 48


The sequence is x 1 x2 x 3 x4 x5
next figure in the series is 24 x5 = 120

ILLUSTRATIVE QUESTION-1

In a certain code ABRACADABRA is coded as ZYIZXZWZYIZ.


How will the word HOCUSPOCUS be coded in that coded
language?
(1) SLXFHKLXFH (2) SLXZHKLXFH
(3) SLXFHKMXFH (4 ) SLXFHILXFH
(5) None of these
Ans. (1) SLXFHKLXFH
Obviously, the alphabet is divided into two groups. For the
first letter from the left end, the first letter from the right end is
substituted as the code and so on.
The principle is as under:A B C D E F G H I J K L M
In the same manner
H OC U S P O C U S
Z Y X WV U T S R Q P O N
SLXFHKLXFH
Therefore, H OC U S P O C U S is coded as SLXFHKLXFH

2.In a certain code language DECEMBER is written as RDEEBCME .


How will FEBRUARY be written in that code language?

(1) YFAERBRU (2) YFREABUR


(3) YFAEBRUR (4 ) YFAERUBR
(5) None of these
Ans. (2) YFREABUR
The alphabets of the given word are numbered from
the left to right and rearranged as follows: 1234 5678
81726 354
DECEMBER

RDEEBCME

1234 5678

81726 354

FE BRUARY

Y F R E A B UR

3.In a certain code language SUCCESS is written as TWFGJYZ .


How will MIRACLE be written in that code language?

(1) NKVEHRKL (2) NKUEHRL


(3) NKTEHRL (4 ) NKUHHRL
(5) None of these
Ans. (2) NKUEHRL

Note that coding is done by replacing each letter in the given word
with its succeeding letter, 2nd succeeding letter, 3rd succeeding letter
and so on. Thus ,

S U C C
E
S
S
- V DE DEF FGHI TUVWX YUVWXY
T W F G
J
Y
Z
Short cut : In such models, ascribing numbers to
the letters of the alphabet would come in handy.

1 2 3 4 5 6 7 8 9 10 11 12 13 14
A B C D E F G H I J K L M N
15 16 17 18 19 20 21 22 23 24 25 26
O P Q R S T U V W X Y Z
MIRACLE is easily coded as :M + 1 = 14 = N
I + 2 = 11 = K
R + 3 = 21 = U
A+4= 5=E
C+5= 8=H
L + 6 = 18 = R
E + 7 = 12 = L
Therefore, MIRACLE = NKUEHRL

4. In a certain code, @ implies divide, * implies subtract, $ implies


multiply and # implies addition. Then, 25$2#9$4@2*5 = ?

(1) 113
(3) 63

(2) 83
(4 ) 93

(5) None of these

Ans. (3) 63
25 $ 2 # 9 $ 4 @ 2 * 5 would be decoded as
25 x 2 + 9 x 4 2 5

= 50 + 9 x 4 5

2
= 50+18 - 5 = 63

ILLUSTRATIVE QUESTION- 5 :- In

a certain code HYD is coded as


50. How will the word NAG be coded using the same
coding system?

(1) 2
(3) 1

(2) 0.5
(4 ) 20 (5) None of these

Ans. (1) 2
Note that in HYD, on numbering, the alphabet from left
to the right with 1 to 26, H = 8, Y = 25 and D = 4.
Values of the first two letters are being multiplied and
then divided by the value of the third letter.
Thus, 8 x 25 = 50

4
In the same way, N = 14, A = 1 and G = 7.
Thus NAG = 14 x 1 = 50 =2

ILLUSTRATIVE QUESTION- 5

2. If in a certain language, MADRAS is coded as NBESBT, how is BOMBAY


coded in that language?
(a)
CPNCBX (b) CPNCBZ (c) CPOCBZ (d) CQOCBZ (e) None of these
Ans. (b) : Each letter of the word is one step behind the corresponding
letter of the code.
3. If FISH is written as EHRG in a certain code, how would JUNGLE be
written in that code?
(a) ITMFKD (b) ITNFKD (c) KVOHMF (d) TJMFKD I
Ans.3. (a : Each letter of the word is one step ahead of the corresponding
letter of the code.
4. In a certain code, TWINKLE is written as SVHOJKD, then how would
FILTE RS be written in the same code?
(a) EHKSDQR (b) EHKUDQR (c) EGKUDQR (d) GJMSFST (e) None of these
5. In a certain code, ROAD is written as URDG. How is SWAN written in that
code? (a) VXDQ (b) VZDQ (c) VZCP (d) UXDQ
6. In a certain code language, OPERATION is written as NODQBUJPO. How
is INVISIBLE written in that code?
(a) JOWJTJCMF (b) JOWJTHAKD (c) HMUHTJCL (d) HMUHTHAKD (e) None
of these

ILLUSTRATIVE QUESTION- 7-11

7, In a certain code, FAVOUR is written as EBUPTS. How is DANGER


written in that CODE?
(a) CBFFDS (b) CBMHDS (c) EBFHDS (d) EBHHFS (E) CBFNAP,
8. If SUMMER is coded as RUNNER, the code for WINTER will be .
(a)
SUITER (b) VIOUER (c) WALKER (d) SUFFER
9. In a certain code, PRODUCTIONS is written as QQPCVEUHPMT. How is
ORIENTATION written in that code?
(a) PQJDOVBSJNO (b) PQJDOUBUJPO (c) PSJFOVBSJ (d)
NSHFMVBSJNO
10. If, in a code, MIND becomes
KGLB and ARGUE becomes YPESC,
then what will be DIAGRAM be in that code?
(a) BGYEPYK (b) BGYPYEK (c) GLPEYKB (d) LKBGYPK (e) None of these
11. In a certain code, BASIC is written as DDULE. How is LEADER written
in that CODE?
(a) NGCFGT (b) NHCGGU (c) OGDFHT (d) OHDGHU

Blood Relation
Detection of relationship between persons are one
of the interesting topics in reasoning of Bank
PO/Clerk examination.
It is required to have thorough knowledge on
blood relations for solving this type of questions.
The following relationships need not be
considered.
They are step-father, stepmother, step-son, stepdaughter, step-brother, step-sister, half brother
and half sister.
Disintegration of statement, puzzles and coded
relations are the different types of problems based
on blood relations.

In these type of questions, a roundabout


complicated description is given in the form of a
certain small relationships and direct relationship
between the person concerned is to be found out.
At this stage, one need to have knowledge of
blood relations and you should try to analyze the
given statements systematically and carefully.
Eg., My mothers only child means myself.
Jayas husbands father-in-laws only daughter
means Jaya (herself).
While solving these questions, read carefully the
given information in the question.
Keep your personal biases perceived notions
aside and solve these questions by any of the two
methods hereafter forthcoming.

In solving problems of Blood Relationships, translation of


the given information in to ones own mother tongue and
substituting ones own relationships with ones owns kith
and kin is a very useful strategy.

Certain problems demand the knowledge of representing


the given statement (s) in the form of a diagram/chart.
In drawing the diagrams / charts there are no hard and fast
rules. You can use your own symbols to depict the
information, but give no room for ambiguity.
Generally, a diagram similar to the Traditional Family Tree
is used.
To be clear about gender, we use m for male and f for
female and x y z etc., for unknown.

One important aspect is that a diagram should


always clearly depict generation
Hence, horizontal lines are drawn to indicate
marriage; vertical lines indicate children, broken
lines for sibling relationships.
Take care of not to misinterpret the data, while
drawing the diagram.
A few generally used notations are :
= marriage relationship
------ = sibling relationship
| or / or \ = children
m = male, M = mother, f = female, F = Father

REMEMBER THESE IMPORTANT POINTS


AFTER READING THE QUESTION, IDENTIFY
THE TWO PERSONS BETWEEN WHOM
RELATIONSHIP IS TO BE ESTABLISHED.
TRY TO CORRELATE THE GIVEN
RELATIONSHIPS.
WHILE CONCLUDING ABOUT THE
RELATIONSHIP BETWEEN THE TWO
PERSONS; BE CAREFUL ABOUT THE
GENDERS OF THE PERSONS INVOLVED.
VARIOUS TYPES OF QUESTIONS ASKED,
DO NOT HAVE MUCH DIFFERENCE,
EXCEPT PATTERN OF QUESTIONS.

SUMMARY OF COMMON BLOOD


RELATIONSHIPS
RELATION

COMMONLY USED WORDS

Mothers or fathers brother

Uncle

Mothers or fathers sister

Aunt

Mothers or fathers father

Grandfather

Mothers or fathers mother

Grandmother

Sons wife

Daughter- in - law

Daughters husband

Son-in-law

Husbands or wifes sister

Sister-in-law

Husbands or wifes brother

Brother-in-law

RELATIONSHIPS.
RELATION

COMMONLY USED WORDS

Brothers or sisters son

Nephew

Brothers or sisters daughter

Niece

Uncles or aunts son or daughter

Cousin

Sisters husband

Brother-in-law

Brothers wife

Sister-in-law

Grand sons or grand daughters


daughter

Great granddaughter

Grand sons or grand daughters


son

Great grandson

Sons son
Sons daughter

Grandson
Granddaughter

RELATIONSHIPS.
RELATION

COMMONLY USED WORDS

Fathers or Mothers son

Brother

Fathers or Mothers only son

My/himself

Fathers or Mothers daughter

Sister

Fathers or Mothers only daughter

My/herself

Fathers brother

Paternal Uncle

Mothers brother

Maternal uncle

Fathers sister

Paternal Aunt

Mothers sister

Maternal Aunt

GENERATIONS.
GENERATION

Names of relations

1st GENERATION

Paternal Grand father,


Paternal Grand mother,
Maternal Grand father and
Maternal Grand mother

2nd

GENERATION

Father, Mother, Uncle (Paternal &


Maternal) Aunt (Paternal &
Maternal)

3rd GENERATION

Yourself, Brother, Sister, Cousin,


Brother-in-law and Sister-in-law

4th GENERATION

Son, daughter, nephew, niece,


Son- in law and
Daughter- in - law

5th GENERATION

Grandson, (son of son or


daughter) granddaughter

ILLUSTRATIVE Q UESTION- 1

If A+B means A is the mother of B; A B means A is the brother


of B; A x B means A is the son of B; and A-B means A is the
sister of B, then find the relation of the following situation.
C-PD=?

1. C is the sister of P and the mother of D


2. C is the sister of P and D
3. C is the brother of D who is the sister of D
4. C is the mother of P and D
5. None of these
Ans. 2. C is the sister of P and D
C-P = C is the sister of P
P D = P is the brother of D
Therefore, C is the sister of P and D
This can be depicted as C(f)----P(m)-----D(x)
Note that the gender of D is unknown and hence, denoted as
x.

ILLUSTRATIVE Q UESTION- 2

A man said to a woman. Your only brothers son is my


wifes brother. How is the woman related to the mans
wife?
1. sister
2. Mother-in-law
3. Aunt
4. Sister-in-law
5. None of these
Ans. 3. Aunt
The information could be deciphered as :
Woman Womans brother
|
|
Mans wife - Womans brothers son
Therefore, the woman is mans wifes fathers sister. Aunt.

ILLUSTRATIVE Q UESTION- 3
Study the following information, and answer
the questions that follow.
There are six persons P,Q,R,S,T and U in a
family. R is the sister of U. Q is the brother
of Ts husband. S is the father of P and
grand father of U. There are two fathers,
three brothers and a mother in the group.
3. Name the group of brothers.
1. Q.R.S 2. U,Q,T 3.Q,P,U 4. P,R,S 5.S.P.U
Ans. Q.P.U

The given information can be depicted as a family tree, looks like


1) R is the sister of U R(f)..U
2) Q is the brother of Ts husband Q(m).Ts husbandT(f)
3)S is the father of P and Grandfather of U. S(m) & P(m)
S(m)
P(m)

U
Also, there are two fathers, three brothers and a mother in the group. It
follows that U is male. Now, on clubbing all the above three
statements, the given data would look alike:
S(m)
Q(m)

P(m)

R(f)

U(m)

T(f)

Thus,
The Two fathers=S and P
The three brothers = Q, P and U
One mother = T

Q. 4. Who is the mother?


(1) S. (2) Q. (3) R. (4) T.
Ans. T
T is the mother of R and U

(5) None of these

Q. Who is Ts husband?
(1)P. (2) Q. (3) U. (4)S. (5) None of these
Ans. P
P is Ts husband.
Q.5. Pointing towards a person a man said to a woman.
His mother is the only daughter of your father. How
is the woman related to that person?

1. Daughter 2. Sister 3.Mother 4. Wife 5.None of


these
Ans. The only daughter of womans father is she
herself. So the person is womans son. ie., the
woman is the persons mother.
Hence, the answer is (3).

PRACTICE QUESTIONS 1-7


1. Pointing to Rajesh, Aruna said, He is the son of my fathers only son. How is
Rajeshs mother related to Aruna?
(a) Daughter (b) Aunt (c)Sister (d) Sister-in-law (e) None of these
2. Introducing Soman, Radha said, He is the only son of my mothers mother.
How is Radha related to Soman?
(a) Mother (b) Aunt (c) Sister (d) Niece (e) None of these
3. Brothers sisters husbands father-in-law is:
(a) Grandfather (b)Uncle (c) brother-in-law (d) father (e) None of these
4. Introducing Anurag to her husband, a woman said His brothers father is the
only son of my grandfather. How is the woman related to Anurag?
(a) Mother (b) Aunt (c) Sister (d) daughter (e) grandmother
5. Pointing to Reghu, Geetha said,His mothers brother is the father of my son
Balu. How is Reghu related to Geetha?
(a) Sister-in-law (b) Nephew (c) Niece (d) Aunt (e) None of these
6. Pointing to a girl on the stage, Govind said, Her mothers brother is the only son
of my mothers father. How is the girls mother related to Govind?
(a) Mother (b) Sister (c)Aunt (d) Grandmother (e) None of these
7. M introduces N saying, He is the husband of the granddaughter of the father of
my father. How is N related to M?
(a) Brother (b) Son (c) Brother-in-law (d) Nephew (e) None of these

ANSWER KEY:PRACTICE QUESTIONS 1-7


1. Pointing to Rajesh, Aruna said, He is the son of my fathers only son. How is
Rajeshs mother related to Aruna?
(a) Daughter (b) Aunt (c)Sister (d) Sister-in-law (e) None of these
2. Introducing Soman, Radha said, He is the only son of my mothers mother.
How is Radha related to Soman?
(a) Mother (b) Aunt (c) Sister (d) Niece (e) None of these
3. Brothers sisters husbands father-in-law is:
(a) Grandfather (b)Uncle (c) brother-in-law (d) father (e) None of these
4. Introducing Anurag to her husband, a woman said His brothers father is the
only son of my grandfather. How is the woman related to Anurag?
(a) Mother (b) Aunt (c) Sister (d) daughter (e) grandmother
5. Pointing to Reghu, Geetha said,His mothers brother is the father of my son
Balu. How is Reghu related to Geetha?
(a) Sister-in-law (b) Nephew (c) Niece (d) Aunt (e) None of these
6. Pointing to a girl on the stage, Govind said, Her mothers brother is the only son
of my mothers father. How is the girls mother related to Govind?
(a) Mother (b) Sister (c) Aunt (d) Grandmother (e) None of these
7. M introduces N saying, He is the husband of the granddaughter of the father of
my father. How is N related to M?
(a) Brother (b) Son (c) Brother-in-law (d) Nephew (e) None of these

PRACTICE QUESTION- 8
8. A is the wife of B and B is the brother of
C. D is the father of C. How is D related
to A?
(a) son (b) daughter (c) Father-in-law (d)
Mother (e) Mother-in-law
Illustration: Draw a family chart.
B is the husband of A
D is the father of A
.: D is the father in law of B

Hence, the answer is: (c) Father-in-law

PRACTICE QUESTION- 9
Read the given information carefully and answer the questions that
follow.
A family consists of 5 members P,Q,R, S and T. P has two sons, one
unmarried daughter and only one daughter- in- law. T is the brother
in-law of the above mentioned daughter- in- law. Qs sister is not
happy with Qs wife. But T and his father support Qs wife S.
Qn.9. Who is the daughter of P?
(a) Q (b) R (c) S (d) T (e) None of these

Illustration: Draw a family chart starting from father P.


Below P there are 4 persons
2 sons, 1daughter, 1
daughter in law
.: R is the daughter of P

Hence, the answer is: (b) R

PRACTICE QUESTION- 10-12


A family consists of 5 members P,Q,R, S and T. P has two sons, one
unmarried daughter and only one daughter- in- law. T is the brother
in-law of the above mentioned daughter- in- law. Qs sister is not
happy with Qs wife. But T and his father support Qs wife S.

Qn.10. How is T related to S?


(a) Sister (b) Brother (c) Sister-in-law (d) Brother- in-law (e)
None of these
Ans. (d) Brother-in-law
Qn.11. How is P related to S?
(a) Sister (b) Brother (c) Sister-in-law (d) Brother- in-law (e)
father-in-law
Ans. (e) Father-in-law
Qn.12. Who is the sister of Q?
(a) Q (b) R (c) S (d) T (e) None of these
Ans. (b) R

Directions: (Q.13 -16) : Read the given information carefully and answer
the questions that follow.

I. A,B,C,D and F are six members of a family.


II. One couple has parents and their children in the family.
III. A is the son of C and E is the daughter of A.
IV. D is the daughter of F who is the mother of E.
Qn.13. Who are the male members in the family?

(a) A and C (b) C and F (c) A,B, and D (d) cannot be determined (e)
None of these
Ans. (d) The sex of B and C cannot be determined

Qn.14. Which of the following pairs is the parents of the children?


(a) BC (b) CF (c) BF (d) cannot be determined (e) None of these

Ans. (e) Clearly A and F are the parents of the children D & E
Qn.15. Which of the following pairs is the parents of the couple?
(a) AB (b) BC (c) AF (d) CF (e) None of these

Ans. (e) Clearly B and C are the parents of the couple


Qn.16. How many female members are there in the family?
(a) Two (b) 3 (c) 4 (d) cannot be determined (e) None of these

Ans. (c) 4 .Clearly female members are either B or C, F,D and E

Sense of Direction

Questions based on directions are one of many analytical


reasoning tests.
These questions consists of a sort of directional puzzles.
The outline describes a situation in the form of a small
passage.
You are required to arrange the scattered information
given in the outline portion of the question.
A successive follow up of directions is formulated and the
candidate has to find the final direction with respect to the
starting point or the distance between two points.
These questions are meant to judge the candidates ability
to trace, follow and sense the directions correctly.
A diagram depicting the given information help to solve
the questions quickly.

ILLUSTRATIVE Q UESTION- 1
Facing towards North Govind walks 30 metre. Then he
turns left and walks 50 metre. Then he turns left and walks
30 metre again. Then, he turns right and walk 20 metre.
How far is he now from the starting position?
(1) 60 m (2) 35 m (3) 70 m (4) 50 m (5) None of these
Ans. 70 m

50 m

ILLUSTRATIVE Q UESTION- 2
A man is standing facing West. He turns 45 in the
clockwise direction and then another 180 in the same
direction. Then he turns 270 in the anti-clockwise
direction. Which direction is he facing now?

(1) South (2) North west (3) West


West (5) None of these

(4) SouthA

Ans. (4) South-West


The man initially faces the direction OA. O
On moving 45 clockwise, he
faces the direction OB. After
turning 180 clockwise, he
Faces direction OC. Finally
On moving 270 in the
anti-clockwise, he faces the
Direction OD which is SW.

45
180

270

ILLUSTRATIVE Q UESTION- 3

One day, Rajesh left home and cycled 15 KM southwards, turned right
and cycled 5 KM. He again turned right and cycled 15 KM and again
turned left and cycled 15 KMs. How many KMs he will have to cycle to
reach his home straight?

(1) 10km (2) 15 km (3) 20k m


these
Ans. 20 km

(4) 25km

(5) None of

ILLUSTRATIVE Q UESTION- 4

A child was looking for his father. He went 90 meters in the East before
turning to his right. He went 20 meters before turning to his right again to
look for his father at his uncles place 30 meters from this point. His father
was not there, he went 100 meters to his north before meeting his father in
the street. How far did the son meet his father from starting point?
(1) 80 m (2) 100 m (3) 140 m

Ans. (2) 100 m

(4) 260 m

(5) None of these

The child walks 30 m from A to B, then


turns right and walks 20 m up to C. Then
turns right and walks 30 m up to D. Finally
he turns right and walks 100m up to E.
AB = 90 m, BF = CD =30m
AF =AB-BF = 60 m
Also DE =100m, DF =BC = 20 m
So, EF =DE-DF =80m
Therefore, the distance from the starting
point A = AE
= AE =\/AF2+EF2 = \/(60)2+(802)
= \/3600 +6400 =\/10000 =100

ILLUSTRATIVE Q UESTION- 5
Two buses start from two opposite points of a
main road, 100 KMs apart. The first bus runs 25
kms and takes a right turn and then runs for 15
kms, it then turns left and runs for another 25 kms
and takes the direction to reach the main road. In
the meantime, due to a minor breakdown the other
bus has run only 30 kms along the main road.
What would the distance between the two buses at
this point? .
(1) 65 km (2) 20 km (3) 40 km
Ans. (2) 20 km

(4) 50km

(5) None of these

LET THE FIRST BUS BE DENOTED AS A AND THE OTHER AS B

The distance traveled by A on the main road


=25+25=50km
The distance traveled by B on the main road = 30km
The starting points are 100 KM apart.
Therefore, the buses are now at 100-(50+30) = 20km
apart.

ILLUSTRATIVE Q UESTION- 6
Directions (1-2): Study the given information carefully and answer the given
questions:
Sunil started walking from point A. He walked 6 meters towards East to reach
point B. From point B he took a right turn and walked 3 meters to reach point C.
From point C he took a right turn and walked 10 meters to reach point D. From
point D he took a left turn and walked 4 meters to reach point E. From point E, he
walked 4 meters East to reach point F. Point G is exactly mid-way between point C
and point D.

1. If Sunil walks 4 meters to the North of point F, how far and in which
direction will he be from point G?
Ans. (1) 1 meter towards East (2) 2 meters towards East (3) 1 meter
towards West (4) 2 meters towards West (5) 1.5 meters towards East

Ans. 1. (3)
Explanation:

2. How far and in which direction is


point F from point A?
Ans. (1) 7 meters towards North (2) 6 meters
towards South (3).8 meters towards South
(4) 6 meters towards North (5) None of
these
2. (5); 7 meters towards South.
Explanation:

Sense of Direction Practice Questions


(Q.1-2):Select the correct answer from the given options of direction or distance or both.

Q.1. Sreeni went 30 km to the North, then he turned West and traveled 20
KM, then he turned South and covered 10 km, then turned East and
covered 20 km. In which direction is he from the starting point?
(a) North (b) South (c) East (d) West
Ans. (a) North

2. Sambu started from his school and walks 12 km towards North. He


turns right and walks another 12 km. He turns right again and walks
12km more and turns to left to walk 6 km. How far he is from his school
and in which direction?
(a) 18km North (b) 12 Km South (c) 18 Km East (d) 12Km west

Ans. (c) 18 Km East

Sense of Direction Practice Questions


(Q.3-4):Select the correct answer from the given options of direction or distance or both.
3. Saheer Walked 20 km North. Then he turned right and walked 20km. Then he
turned right and walks 25 km. Then he turned left and walked 20 km. Finally he
turned left and walked 25km. In which direction and how many kms is he from
the starting point?
(a) 40km North (b) 60 Km East (c) 40 Km West (d) 60 Km South
Ans. (b) 60 Km East

4. Chinchu walked 5km towards East and then turned left and walked 6km. Then
she turned right and walks 10km. Finally she turned to her right and covered
6km. How far and in which direction is she from her starting point now?
(a) I5 km North (b) 15 km East (c)I5 Km South (d) I5 Km West

Ans. (b) 15 Km East

Sense of Direction Practice Questions


(Q.5-6):Select the correct answer from the given options of direction or distance or both.
5. A lady walks northwards. After a while she turns towards her right and
proceeds further towards left. Finally she turns to her left again. In which
direction is she moving now?
(a) North (b) South (c) East (d) West (e) South west
Ans. (d) West

6. John went to East up to 80 km., then he turned right and went 50 km. Then
turned right and covered 60 km. Finally he turned right and covered 50 km. How
far is he from the starting point?
(a) 50km (b) 25km (c) 20km (d) 60km (e) 40 km

Ans. (c) 20 Km

Sense of Direction Practice Questions


(Q.7-8):Select the correct answer from the given options of direction or distance or both.
Q.7. Rahim went 30 km towards East. Then he turns to his right and walked
20km. He again turns right and walked 30 km. How far and in which direction is
he from his starting point?
(a) 20km South (b) 20km North (c) 10 km South (d) 10 km North

Ans. (a) 20 KM South


Q.8. Thomas walked 4 km towards East and then turned right and walked 3 km.
Again he turned to his right and walked 3km. Again he turned to his right and
walked 3 km. Again he turned to his right and walked 4 km. After this, he turned
back 2 km. Which direction was he facing then?
(a) North (b) East (c) South (d) West

Ans. (c) South

Sense of Direction Practice Questions


(Q.9-10):Select the correct answer from the given options of direction or distance
or both.
9. Jain is 50 km south-West of Binu. If Vinay is 50 km South- East of Binu, then
Vinay is in which direction of Jain?
(a) North (b) South (c) East (d) West

Ans. (c) East


10. Sandhya walked 50 metres straight from her house which is facing North and
then walked 100 metres in the reverse direction. She then turned to her left
and covered 50 metres, In which direction is she now from her original
position?
(a) South (b) South East (c) North (d) North-West

Ans. (b) South - East

Sense of Direction Practice Questions


(Q.11-12):Select the correct answer from the given options of direction or distance
or both.
11. Seena started from her house, and covered 4km in the East direction, then
she turned to her right and Covered 3 km. What minimum distance will be
covered by her to come back to her house?
(a) 8 km (b) 4 km (c) 5 km (d) 6 km

Ans. (c) 5 KM

12. A person started from his house and walks 100 metres towards South. He
then turned left and walked 75 metres towards East. How far he is from the
starting point?
(a) 75m (b) 100m (c)125m (d)150m

Ans. (c) 125 m

Sense of Direction Practice Questions


(Q.13-14):Select the correct answer from the given options of direction or distance
or both.
13. Jameela started from her house, walked 2 km towards North, then 3 km
towards West and finally 6km towards South. The shortest distance between
her house and the ending point is...
(a) 4 km (b) 5 km (c) 6 km (d) 7 km (e) 8 KM

Ans. (b) 5 KM

14. Anagha went to the college and started from house which is in the East and
comes to a crossing. The road to her left ends in a temple straight ahead lies
the Beach. In which direction is the college if all the four are in different
directions.
(a) East (b) West (c) South (d) North

Ans. (d) North

Sense of Direction Practice Questions


(Q.15-16):Select the correct answer from the given options of direction or distance
or both.
15. Subhash is walking in the East direction. He turns 45o left and then 90o right.
In which direction is he now?
(a) North (b) East (c) North East (d) South West

Ans. (c) North East

16. James walked 3 km towards south and turned right. After walking 2 km he
turned to the left and walked 3km. In which direction was he from the starting
point?
(a) South East (b) North-East (c) South -West (d) North-West

Ans. (c) South West

Sense of Direction Practice Questions


(Q.17-18):Select the correct answer from the given options of direction or distance
or both.

17. Noushad is travelling towards North. Which of the following direction


should he take in order to travel towards south?
(a) Left, Right, Left, Left (b) Left, Left, Right, Right (c) Left, Left, Left,
Left (d) None of these

Ans. 17. (a) Left, Right, Left, Left

18. Alisha is walking towards East. What direction she should follow if
she should walk towards south?
(a) Right, Right, Left, Left, Left (b) Right, Left, Right, Left, Right (c) Right,
Right, Left, Right, Right (d) None of these

Ans. 18. (b) Right, Left, Right, Left, Right

Sense of Direction Practice Questions


(Q.19-20):Select the correct answer from the given options of direction or distance
or both.

19. Sunny is facing towards West. He turns left and covers 10 km and
turns left again and covers 10 km. Then he travels 40km towards right.
He then again turns right and covered 30km. Lastly, he turns right and
covers 50 km. How far he is from the starting point?
(a)45Km (b)35Km (c)30km (d)20km

Ans. 19. (d)2Okm


20. Ajith runs 12 km towards West, 3km towards South and then 8km
towards East. In which direction is he from the starting point?
(a) North-East (b) North-West (c) South -West (d) South East

Ans. 20. (c) South -West

IN THESE TYPES OF QUESTIONS, CLUES ARE GIVEN REGARDING


COMPARISONS AMONG A SET OF PERSONS OR THINGS WITH RESPECT TO
ONE OR MORE QUALITIES.
THE CANDIDATE HAS TO ANALYZE THE WHOLE INFORMATION AND
ESTABLISH A PROPER ASCENDING OR DESCENDING ORDER AND ANSWER
THE QUESTIONS ACCORDINGLY.
Sometimes, questions may appear very simple but they are made tricky by
infusing intricacies and complicated data.
Sometimes, you may feel that the given information is insufficient.
But a cogent and coherent analysis of it provides useful clues.
Sometimes, some superfluous statements may be provided to elude you.
Various types of questions can be asked on this.
In one of the types, you are asked find the rank of a person either from top or
bottom in a class /group.
Sometimes, the ranks of persons from either ends are given and you are asked
to find the total number of persons.
In another type questions are based on ranking; you are required to ascertain
some parameters like height, weight, marks, age etc.
To solve such types of questions, you should use some symbols and notations
to organize THE GIVEN INFORMATION so that you can use the data more
conveniently and quickly.

ILLUSTRATIVE QUESTION- 1

Vijay ranks 16th from top and 49th from bottom in a class. How many
students are there in the class?

(1) 66 (2) 65 (3) 64 (4) Cannot be determined (5) None


of these
Ans. (3) 64
In order to calculate the total number of students in the
class, we add the ranks of students from either end and
then subtract 1.
Total number of students in the class = (16+49)-1 = 64
The position of Vijay is in the class is as shown below:
Top
Bottom
15 students
48 students

ILLUSTRATIVE Q UESTION-2
In a class of 39 students, the ratio of boys to girls is 2 : 1. Reshma ranks 15 th
among all the students from the top and 8th among the girls from bottom.
How many boys are there below Reshma?
(1) 16 (2) 17 (3) 15 (4) Data inadequate (5) None of these

Ans. (2) 17
The ratio of Boys to Girls = 2:1 in a class 39 students.
Therefore, the number of boys

= 39 x 2 = 26
3

Number of Girls

= 39-26 = 13

According to the question,

14 students

15th

24students

7 Girls
Out of 24 students who are below Reshma 7 are Girls. Therefore,
number of Boys who are below Reshma = 24-7 = 17

ILLUSTRATIVE Q UESTION-3

In a row of girls, Geetha is 15th from left and Vaisali is 7th from the right.
On interchanging their places, Vaisali becomes 15th from right. How
many girls are there in the row?

(1) 27 (2) 28 (3) 29 (4) Data inadequate (5) None of these

Ans. (3) 29

Left
14 Girls

Right
15th

6 Girls

Left
14 Girls

Right

15th

6 Girls

On interchanging their places Vaisali is 15th from the right and 15th from left.
So, there are 14 girls ahead of her and 14 Girls behind her.
Therefore, total number of girls = 15+14 = 29

ILLUSTRATIVE QUESTION-4
In a class of 64 students, Sameer ranks 19 th
from the. What is his rank from the bottom?
(1) 46th (2) 45th (3) 44th (4) Data inadequate
(5) None of these
Ans. (1) 46th
According to the question,
Top
18 students
19th
46th

45 students

= (64-19)+1 = 46th
Therefore, Sameer ranks 46th from the bottom.

ILLUSTRATIVE QUESTION- 5
Reema is elder than Diana. Sunil is younger than
Reema and Rohini. Rohini is younger than Diana.
Who among them is the youngest?
(1) Reema (2) Rohini (3) Diana (4) Sunil (5)
Data inadequate
Ans. (4) Sunil
Reema > Diana..(i)
Reema, Rohini > Sunil(ii)
Diana> Rohini..(iii)
From (i), (ii) and (iii) ;
Reema > Diana > Rohini > Sunil

So, Sunil is the youngest.

Directions (6-8): Study the given information carefully and answer the
given questions:
In a group of six people, P. Q, R, S. T and U, each having a different
weight, S is heavier than Q. R is lighter than only T and P. Q is not the
lightest. P is not the heaviest.
6. Who amongst the following is the lightest?
S
(1)T (2)P (3)R (4) U (5) None of these

Ans. (4) U
7. Who amongst the following is the second heaviest?
(1) R (2)S (3)U (4) T (5)P

Ans. (5) P
8. How many people are lighter than S?
(1) None (2) Three (3) Two (4) More than three (5) One

Ans. (3) TWO

Q
T
P
R

Clues: R is
lighter than
only T and P.
Q is not the
lightest.
P is not the
heaviest.
S
Q
U

EXPLANATION

T
P
R
S
Q
U

HEAVIEST

LIGHTEST

Hence, Answers are for Q6.(4);


Q7. (5);
Q8. (3);

Directions (9). Fifteen girls are standing in a straight line facing North.
Sudha, is standing eleventh from the right end. Radha is standing ninth
from the left end. Meena is standing exactly between Sudha and Radha.
How many girls are standing to the right of Meena?

(1) Eight (2) Seven (3) Nine (4) Six (5) Cannot be determined

4 excluding Sudha (5 from L), Meena 7th, Radha 9th

11 +1=12 including Sudha

Directions (10-14): Study the given information


carefully and answer the given questions:

Eight people P, Q, R, S, T, U, V and W are sitting around a circular table facing


the centre not necessarily in the same order. T is sitting third to the right of P.
W is sitting second to the right of S. S is not an immediate neighbour of either P
or T. U and Q are immediate neighbours of each other. Q is not an immediate
neighbour of P. V is not an immediate neighbour of W.
10. What is Rs position with respect to V?
(1) Second to the left (2) Second to the right (3) Third to the right (4)
Third to the left (5) None of these
11. Four of the following five are alike in a certain way based on their
seating positions in the above arrangement and so form a group.
Which one does not belong to the group?
(l) RQ (2) PV (3) TP (4) US (5) WT
12. Who is sitting second to the right of the one who is sitting to the
immediate right of W?
(1) U (2) R (3) V (4) P (5) Q
13. How many people are sitting between U and W when counted from the
left side of U?
(1) Three (2) None (3) More than three (4) Two (5) One
14. Who is sitting exactly between T and S?
(l) U (2)Q (3)W (4)V (5)R

Explanation

10. What is Rs position with respect to V?


Ans. (2) Second to the right
11. Four of the following five are alike in a certain
way based on their seating positions in the
above arrangement and so form a group.
Which one does not belong to the group?
Ans. (l) RQ (2) PV (3) TP (4) US (5) WT
12. Who is sitting second to the right of the one
who is sitting to the immediate right of W?
Ans. (1) U (2) R (3) V (4) P (5) Q

13. How many people are sitting


between U and W when counted
from the left side of U?
Ans.(1) Three (2) None (3) More
than three (4) Two (5) One
14. Who is sitting exactly between
T and S?
Ans.(l) U (2)Q (3)W (4)V (5)R

Directions (15-19) : Study the given information carefully and


answer the given questions:

Eight people A, B, C, D, E, F, G and H are sitting in a straight line facing North


not necessarily in the same order. F is sitting fifth to the right of A. D is sitting
fifth to the right of E. E is to the immediate right of A. H is third to the left of D. B
is third to the right of G. G is an immediate neighbour of A.
15. What is Es position with respect to C?
(1) Second to the left (2) Third to the right (3) Fourth to the right (4) Third to the
left (5) None of these
16. Who is sitting third to the right of the one who is sitting sixth from the right end
of the above arrangement?
(l) D (2) C (3) E (4) H (5) None of these
17. Who amongst the following are sitting at the extreme ends of the line?
(l) G and D (2) A and D (3 ) G and F (4) E and D (5) A and B
18. Four of the following five are alike in a certain way based on their seating
positions in the above arrangement and so form a group. Which one does not
belong to the group?
(l)HD (2)BE (3)EC (4)AH (5)GB
19. If all eight persons are asked to sit in an alphabetical order from right to left,
the positions of how many will remain unchanged as compared to their original
seating position?
(1) Three (2) More than three (3) One (4) Two (5) None

Explanation

1 2 3 4 5 6 7 8

15. What is Es position with respect to C?


(1) Second to the left (2) Third to the right (3) Fourth to the right (4) Third to the left (5) None
of these
Ans. (4); Third to the left
16. Who is sitting third to the right of the one who is sitting sixth from the right end of the
above arrangement?
(l) D (2) C (3) E (4) H (5) None of these
Ans.16. (2); C
17. Who amongst the following are sitting at the extreme ends of the line?
(l) G and D (2) A and D (3 ) G and F (4) E and D (5) A and B
Ans.17.(1); (l) G and D
18. Four of the following five are alike in a certain way based on their seating positions in the
above arrangement and so form a group. Which one does not belong to the group? (l)HD
(2)BE (3)EC (4)AH (5)GB Ans.18.(2); BE
19. If all eight persons are asked to sit in an alphabetical order from right to left, the
positions of how many will remain unchanged as compared to their original seating
position? (1) Three (2) More than three (3) One (4) Two (5) None

VERBAL REASONING

These types of questions are comparatively simple


but require sharp reasoning to arrive at the
answer.

ILLUSTRATIVE QUESTION-1
1. How many pairs of letters are there in the word BUCKET which
have as many letters between them in the word as in the alphabet?

(1) One (2) Two (3) Three (4) Four


(5) More than four
Ans. (1) One
BUCKET letters in word

CDE

- Letters in the Alphabet

ILLUSTRATIVE QUESTION-2
2. How many pairs of letters are there in the word
BRIGHTEN each of which has as many letters
between them in the word as in the English
Alphabet?

(1) None (2) One (3) Two


Three (5) More than three
Ans. (3) TWO
BRIGHTEN letters in word
IJKLMN
GH

- Letters in the Alphabet

(4)

ILLUSTRATIVE QUESTION-3
Two letters in the word PRESENSE have
as many letters in them in the word as in
the alphabet and in the same order. Which
among the two letters comes earlier in the
alphabet?
(1) C (2) E (3) R (4) P (5) None of these
Ans. (4) P

PRESENSE in the word


PQRS - in the alphabet
Between P and S, P comes first

ILLUSTRATIVE QUESTION-4
From the word LAPROSCOPY, how many independent
meaningful words can be made without changing the order
of the letters and using each letter only once?(

1) One (2) Two (3) Three (4)


Four (5) More than four
Ans. (2) Two
The words that can be made from
LAPROSCOPY are LA, SCOPY or
LAP, SCOPY or LAP, COPY

ILLUSTRATIVE QUESTION-5
Which letter in the word
SELFRIGHTEOUSNESS, does not change
its position when the letters are reversed?
(1) E (2) G (3) H (4) T (5) None of these
Ans. (4) T
When the letters in the word are written in
the reverse order, the position of the middle
letter remain unchanged.
Thus in the word SELFRIGHTEOUSNESS,
the middle letter T does not change its
position when the letters are reversed.

ILLUSTRATIVE QUESTION-6
If the first and second letters in the word
DEPRESSION are interchanged, also the third
and fourth letters, the fifth and sixth letters and so
on, which of the following would be the seventh
letter from the right?
(1) R (2) O (3) S (4) I (5) None of these
Ans. (5) None of these
Given word : DEPRESSION

On interchanging the letters: EDRPSEISNO


The seventh letter from the right is P which is not
there in the answer options. Hence 5 is the
answer.

ILLUSTRATIVE QUESTION-7
If the first and fourth letters of the word SENIOR are
interchanged, also the second and fifth letters and the third and
sixth letters, how many letters are there between the letter which
is second from right and the letter which is first from the left, in the
English alphabetical order?

(1) None (2) One (3) Two (4) Three (5) More than
three
Ans. (4) Three
Given word : SENIOR
On interchanging the letters: IORSEN

The required letters are E and I [ E(FGH)I] from the


right, I and N [I(JKLM)N] and R and S which is first from
the left in the alphabetical order.

Hence (4) is the answer.

ILLUSTRATIVE QUESTION-8
Directions: (Qns. 1 - 5) The following questions are based on five
words given below.
WAX NET FOR HUB LIE

(The new words formed after performing the mentioned


operations may or may not necessarily be meaningful
English words)
1. If the positions of first and third letters of each of the words are
interchanged, which of the following will form a meaningful
English word?
(1) Both HUB and WAX (2) LIE (3) Both FOR and LIE (4) WAX (5)
NET
Ans. (5) NET
WAX XAW , NET - TEN , FOR ROF, HUB BUH, LIE EIL
Out of these, NET TEN is a meaningful word
Hence, the answer is (5)

QUESTION-82..3
If the given words are arranged in the order as they would
appear in a dictionary from left to right, which of the following
will be the fourth from the right ?

(1) FOR (2) HUB (3) LIE (4) NET (5) WAX
Ans. (2) HUB
Arranging as required : FOR, HUB, LIE, NET, WAX
Now fourth from right is HUB

Hence (2) is the answer.


Qn.3. If in each of the given words, each of the consonants changed to
previous letter and each vowel is changed to next letter in English
alphabetical series, in how many words thus formed will no vowels
appear?

(1)None (2) One (3) Two (4) Three (5) More than three
Ans. (4) Three
WAX VBW, NET MFS, FOR EPO, HUB GVA, LIE - KJF

QUESTION-845
4. If the first alphabet in each of the words is changed to next
alphabet in the English alphabetical order, how many words
having two vowels (same or different vowels) will be there ?

(1)None (2) One (3) Two (4) Three (5) four


Ans. (4) Three
Arranging as required : WAX - XAX, NET - OET, FOR GOR,
HUB IUB, LIE - MIE
Words formed having two vowels : OET, IUB, MIE

Hence (4) is the answer.


Qn.5. How many letters are there in the English alphabetical order,
between the third letter of the word which is third from the right and
third letter of the word which is second from the left of the given
words?

(1) one (2) Two (3) Three (4) four (5) five
Ans. (1) One : R,T

Classification

CLASSIFICATION MEANS TO ASSORT THE ITEMS OF A GIVEN


GROUP ON THE BASIS OF CERTAIN COMMON QUALITY THEY
POSSESS AND THEN SPOT THE ODD ONE OUT.

Letter based classifications can be in the following three


forms:
a) Based on letters b) Based on letter groups and (c)
Based on letter arrangements
In this test, you are given a group of certain items, out of
which all except one are similar to one another in some
manner.
The candidate is required to choose this one item which
does not fit into the given group.
DIFFERENT TYPES OF QUESTIONS THAT CAN BE ASKED ARE AS
FOLLOWS: (1) CHOOSE THE ODD WORD OUT
IN THESE QUESTIONS, YOU ARE GIVEN A GROUP OF CERTAIN ITEMS
OUT OF WHICH ALL EXCEPT ONE ARE SIMILAR TO ONE ANOTHER IN
SOME MANNER.
YOU HAVE TO IDENTIFY THE ITEM WHICH IS NOT FIT INTO THE GIVEN
GROUP.

ILLUSTRATIVE QUESTIONS
Directions: Choose the word which is least like the other words
in the group.
Qn.No.1. (a) Curd (b) Butter (c) Oil (d) Cheese (e) Cream
Solution:
Here all except oil are products obtained from Milk and from a
class or group.
Hence, the odd word is oil and therefore the answer is (c).
Qn.No.2. (a) Rose (b) Lotus (c) Marigold (d) Ixora (e) Lily
Solution:
Here all except Lotus are flower plants, which grow on land
and form into a class or group.
Hence, the odd word is Lotus and therefore the answer is (b).
Qn.No.3. (a) Pistol (b) Sword (c) Gun (d) Rifle (e) Cannon
Solution: Here all except Sword are fire arms and form a
class or group.
Hence, the odd word is Sword and therefore the answer is (b).

ILLUSTRATIVE QUESTIONS
Directions: In each of the following questions, five words have
been given, out of which four are alike in some manner and the
fifth one is different. Choose the word which is least like the
other words in the group.
Qn.No.4. (a) Tortoise (b) Duck (c) Snake (d) Whale (e) Crow
Solution:
Here all except Whale lay eggs and form in to a class or group.
Hence, the odd word is Whale and therefore the answer is (d).
Qn.No.5. (a) Uncle (b) Nephew (c) Brother (d) Cousin (e) Niece
Solution:
Here all except Brothers are relations based on parents,
brothers and sisters.
Hence, the odd word is Brothers and therefore the answer is (c).
Qn.No.6. (a) Reader (b) Writer (c) Publisher (d) Printer (e) Reporter
Solution: Here all except Reader are persons involved in the
preparation of a journal, news paper or magazine and form a
class or group.
Hence, the odd word is Reader and therefore the answer is (a).

ILLUSTRATIVE QUESTIONS
Directions: In each of the following questions, find out the group of
letters which is different from others.
Qn.No.7. (a) ABC (b) OPQ (c) UVW (d) HIJ (e) EFG
Solution:
Here all except HIJ the group or class is formed with a vowel at
the beginning.
Hence, the odd word is HIJ and therefore the answer is (d).
Qn.No.8. (a) R9 (b) N14 (c) M14 (d) J17 (e) Z1
Solution:
Here all except N14 are relations based on letter and its number
position from right. N14 is in neutral order.
Hence, the odd word is N14 and therefore the answer is (b).
Qn.No.9. (a) JKIJ (b) RSQR (c) BDCB (d) VWUV (e) MNLM
Solution: Here all except BDCB are in the pattern =1,-2,+1 (ie.,
J+1=K, K-2=I, I+1=J.
So, JKIJ
HENCE THE ANSWER IS (C)
Q.no.10. (a) ADE (b) FGE (c) POS (d) UVW (e) IJK
Solution: Here all except (a) are in the pattern one vowel +2
consonants So, the answer is (a) ADE

IN THESE TYPE OF QUESTIONS, YOU ARE


GIVEN CERTAIN PAIRS OF WORDS OUT OF
WHICH THE WORDS IN ALL THE PAIRS
EXCEPT ONE, BEAR A COMMON
RELATIONSHIP.
YOU HAVE TO DECIPHER THE
RELATIONSHIP AND CHOOSE THE PAIR IN
WHICH THE WORDS ARE DIFFERENTLY
RELATED THAN THE REMAINING PAIRS, AS
THE ANSWER .

ILLUSTRATIVE QUESTIONS
Directions: In each of the following questions, five pairs of words have
been given, out of which four pairs bear a certain common relationship
in some manner and the fifth pair is different. Choose the pair in which
the words are differently related.
Qn.No.1. (a) Mason : Wall (b) Cobbler : Shoe (c) Farmer : Crop (d) Chef :
Cook (e) Choreographer : Ballet
Solution:
Here all except Chef : Cook, second is prepared by the first and form in
to a class or group.
Hence, the odd pair is Chef : Cook and therefore the answer option is
(d).
Qn.No.2. (a) Blacksmith : Anvil (b) Carpenter : Saw (c) Goldsmith :
ornaments (d) Barber : Scissors (e)Sculptor : Chisel
Solution:
Here all pairs except Goldsmith : Ornaments second is the tool used
by the first.
Hence, the answer is (c).
Qn.No.3. (a) Pascal : Pressure (b) Watt : Power (c) Ampere : Current (d)
Radian : Degree (e) Joule : Energy
Solution: Here all pairs except Radian : Degree first is a unit to measure
the second and form a class or group.
Hence, the odd pair is Radian : Degree and therefore the answer is (d).

IN THESE TYPE OF QUESTIONS, YOU ARE GIVEN CERTAIN


NUMBERS OUT OF WHICH ALL EXCEPT ONE, ARE ALIKE IN
SOME MANNER WHILE ONE IS DIFFERENT.
YOU HAVE TO IDENTIFY THIS NUMBER AND THE NUMBER
IS TO BE CHOOSEN AS THE ANSWER.
Ex.1. (a) 12 (b) 25 (c) 37 (d) 49 (e) 57
Here, it is evident that 37 is the only prime number in this
group.
Hence, the answer is (c).
Ex.2. (a) 8314 (b) 2709 (c) 1315 (d) 2518 (e) 3249
Here, it is evident that in all numbers except 8314, the sum of first three
digits is equal to the units digit.
Hence, the answer is (a).
Ex.3. (a) 48 (b) 12 (c) 36 (d) 24 (e) 59
Here, it is evident that in all numbers except 59, the units digits is twice
the tens digit.

Hence, the answer is (e).

Choose the odd numeralPractice questions


Directions: In each of the following questions, five numbers
are given. Out of these, four are alike in a certain manner
but the fifth one is different. Choose the one in which is
different from the rest four.
1. (a) 43 (b) 53 (c) 63 (d) 73 (e) 83
2. (a) 10 (b) 26 (c) 24 (d) 21 (e) 18
3. (a) 51 (b) 144 (c) 64 (d) 121 (e) 256
4. (a) 15 (b) 21(c) 24 (d) 28 (e) 30
5. (a) 324 (b) 244 (c) 136 (d) 352 (e) 514
6. (a) 6 (b) 12 (c) 18 (d) 9 (e) 7
7. (a) 45 (b) 99 (c) 109 (d) 126 (e) 207
8. (a) 27 (b) 125 (c) 343 (d) 729 (e) 1321
9. (a) 21 (b) 39(c) 51 (d) 63 (e) 83
10. (a) 35 (b) 49 (c) 50 (d) 63 (e) 140
11. (a) 385 (b) 572 (c) 671 (d) 264 (e) 427
12. (a) 2384 (b) 1592 (c) 3756 (d) 4298 (e) 3629

Answer Key to Practice questions


1. (a) 43 (b) 53 (c) 63 (d) 73 (e) 83
Sol. (c) : Each of the numbers except 63, is a prime number.
2. (a) 10 (b) 26 (c) 24 (d) 21 (e) 18
Sol. (d) : Each of the numbers except 21, is an even number.
3. (a) 51 (b) 144 (c) 64 (d) 121 (e) 256
Sol. (a) : Each of the numbers except 51, is a perfect square.
4. (a) 15 (b) 21(c) 24 (d) 28 (e) 30
Sol. (d) : Each of the numbers except 28, is divisible by 3.
5. (a) 324 (b) 244 (c) 136 (d) 352 (e) 514
Sol. (a) : Sum of the digits in each other number is 10.
6. (a) 6 (b) 12 (c) 18 (d) 9 (e) 7
Sol. (e) : 7, is the only prime number in the group.
7. (a) 45 (b) 99 (c) 109 (d) 126 (e) 207
Sol. (c) : Each of the numbers except 109, is divisible by 9.
8. (a) 27 (b) 125 (c) 343 (d) 729 (e) 1321
Sol. (e) : All other numbers cubes of odd numbers.
9. (a) 21 (b) 39(c) 51 (d) 63 (e) 83
Sol. (e) : 83, is the only prime number in the group.
10. (a) 35 (b) 49 (c) 50 (d) 63 (e) 140
Sol. (c) : Each of the numbers except 50, is divisible by 7.
11. (a) 385 (b) 572 (c) 671 (d) 264 (e) 427
Sol. (e) : In all other numbers except 427, the middle digit is the sum of the other two.
12. (a) 2384 (b) 1592 (c) 3756 (d) 4298 (e) 3629
Sol. (e) : In all other numbers except 3629, the last digit is two times the first.

IN THESE TYPE OF QUESTIONS, YOU ARE GIVEN CERTAIN


PAIRS OR GROUPS OF NUMBERS OUT OF WHICH ALL
EXCEPT ONE ARE SIMILAR IN SOME MANNER WHILE ONE
IS DIFFERENT.
THE NUMBERS IN THESE SIMILAR PAIRS MAY HAVE THE
SAME PROPERTY OR MAY BE RELATED TO EACH OTHER
ACCORDING TO THE SAME RULE.
YOU HAVE TO CHOOSE THE ODD PAIR/GROUP.

Ex.1. (a) 83-75 (b) 58-50 (c) 49-42 (d) 25-17 (e) 57- 49
Here, it is evident that in each pairs except (c), the first
number is 8 more than the second.
Hence, the answer is (c).
Ex.2. (a) 70-80 (b) 54-62 (c) 28-32 (d) 21-24 (e) 14-16
Here, it is evident that in each pairs except (b), the ratio of
the two numbers is 7:8.
Hence, the answer is (b).

What is Analogy?
Analogy is a form of reasoning in which a similarity between two or more
things is inferred from a known relationship between them. As there can
be more than one relationship between two objects, it is essential to
understand which relationship the examiner wants the candidate to
locate in a particular question.
IN OTHER WORDS, ANALOGY MEANS CORRESPONDENCE.
Look at the following pair:
Ocean: Island
The pair can have the following possible relationships:
(i) The first surrounding the second. (ii) The first, a place to swim; the second
to walk. (iii) The second being very small in comparison with the first. (iv)
Specific names of ocean and island located in it.
On the basis of these relationships we can liken this pair to the following pairs
respectively
Protoplasm : Nucleus (ii). Water : Soil iii) Desert : Oasis iv) Pacific : Greenland
Or Indian : Nicobar
Thus the choice of given pairs would determine which relationship to highlight.
In case of completing the analogous pair, the word given would provide the
hint for the relationship we have to look for.

Analogy y is a verbal part of the Psychological and


Reasoning Tests and forms an important
ingredient of many competitive examinations.
VERBAL ANALOGICAL TESTS USE WORDS LETTERS AND
DIGITS .
REQUIRE COMMONSENSE REASONING TO SOLVE
QUESTIONS BASED ON THESE.
IN THESE QUESTIONS, A PARTICULAR RELATIONSHIP IS
GIVEN AND ANOTHER SIMILAR RELATIONSHIP HAS TO
BE IDENTIFIED FROM THE ALTERNATIVES PROVIDED.
THESE QUESTIONS TEST THE CANDIDATES OVERALL
KNOWLEDGE AND ABILITY TO THINK CONCISELY AND
ACCURATELY.

DIFFERENT TYPES OF QUESTIONS THAT CAN BE


ASKED ON ANALOGY ARE AS FOLLOWS
1. COMPLETING THE ANALOGOUS PAIR:
In these two words related to each other are given along
with a third word.
You have to find the relationship between the first two
words and choose the words from the given options that
bears the same relationship with the third word as the first
two words.
Two examples are given below:
1. Ship : Propeller :: Car : ?
(a) Wheel (b) Accelerator (c) Petrol (d) Steering
Explanation: (b) The second moves the first.
2. Loud : Stentorian :: Bright : ?
(a) Resplendent (b) Dull (c) Sunshine (d) Car
Explanation: (a) The second is heightened form of the first.

Directions (Qs. No. 1 to 20) : In each of the following questions the Two words
given on one side of double colon i.e., :: bear a certain relationship. The other
side shows an incomplete pair marked by a missing word which bears a similar
relationship with the given word. Find the missing word out of the given options
(a) (b) (c) or (d). Check your scores with Answers and Explanations.
1. King : Royal :: ? : Religious
(a) Prayer (b) Saint (c) Priesthood (d) Holy Boop
2. Hill : Mountain :: ? : Pain
(a) Distress (b) Discomfort (c) Headache (D) Fear
3. Raipur : Chhatisgarh:: ? : Uttaranchal
(a) Nainital (b) Ranchi (c) Dehradun (d) Bhopal
4. Eye : Cataract :: ? : Hepatitis
(a) Liver (b) Pancreas (c) Stomach (d) Lungs
5. Oxygen : Burn :: ? : Extinguish
(a) Nitrogen (b) Helium (c) Hydrogen (d) Carbon
dioxide
6. Sculptor : Statue :: ? : Poem
(a) Painter (b) Writer (c) Singer
(d) Poet
7. Jockey : Horse : : ? : Car
(a) Chauffeur (b) Tyre (c) Engine (d) Driver
8. Reading : Knowledge :: ? : Experience
(a) Job (b) Work (c) Traveler (d) Training
9. Stomata : Leaf :: ? : Bark
(a) Pore (b) Cell (c) Lenticels (d) Stem
10. Autotroph : Producer :: ? : Consumer
(a) Heterotroph (b) Carnivore (c) Herbivore (d)
Entrepreneur

11. Potato : Stem :: Radish : ?


(a) Leaf (b) Root (c) Flower (d) Seed
12. Paradigm : Pattern :: Method : ?
(a)Decoration (b) Weaving (c)System(d)print
13. Brick : Building :: ? : Letter
(a) Phrase (b) Word (c) Ink (d) Alphabet.
14. Computer : E-mail :: Mobile : ?
(a) Fax (b) SMS (c) Call (d) Ring
15. Axis : Earth :: ? : Wheel
(a) Equinox (b) Tyre (c) Hub (d) Gear
16. Reluctant : Keen :: Remarkable : ?
(a) Enthusiastic (b) Usual (c) Splendid (d)
Talented
17.Mumbai:Maharashtra::Thiruvananthapuram:?

(a) Mangalore (b) Kochi (c) Nilgiri (d) Kerala


18. Herd : Sheep :: ? : Fish
(a) Gale (b) Shoal (c) Basket. (d) Aquarium
19. Accommodation : Rent :: Travel : ?
(a) Fare (b) Ticket (c) Pass (d) Charges
20. Line : Square :: Arc : ?
(a) Semicircle (b) Circle (c) Triangle (d) Line

2. CHOOSING THE ANALOGOUS PAIR


IN THESE TYPE OF QUESTIONS, A PAIR OF WORDS IS
GIVEN, FOLLOWED BY FOUR PAIRS OF WORDS AS
OPTIONS.

YOU HAVE TO CHOOSE THE PAIR IN WHICH THE


WORDS HAVE THE SAME RELATIONSHIP TO
EACH OTHER AS THE RELATIONSHIP IN THE
GIVEN PAIR.
Ex.1. Darkness: Lamp
(a) Fatigue : Exercise (b) Thirst: Water (c) Medicine:
Illness (d) Study: Classroom (e) Leg : Socks
Sol. Just as a lamp eliminates darkness, water
eliminates thirst.
Hence, the answer is (b)

3. THREE WORD ANALOGY


IN THESE TYPE OF QUESTIONS, A GROUP
OF THREE INTER-RELATED WORDS IS
GIVEN.

YOU HAVE TO DECIPHER THE


RELATIONSHIP AMONG THESE YHREE
WORDS AND CHOOSE ANOTHER
GROUP WITH SIMILAR ANALOGY
FROM THE ALTERNATIVES THAT ARE
PROVIDED.

4. NUMBER ANALOGY
TWO TYPES OF QUESTIONS ARE ASKED ON
THIS.
1. CHOOSING A SIMILARLY RELATED PAIR
AS THE GIVEN NUMBER PAIR ON THE BASIS
OF RELATION BETWEEN THE NUMBERS IN
EACH PAIR.
2. CHOOSING A NUMBER SIMILAR TO A
GROUP OF NUMBERS ON THE BASIS OF
CERTAIN COMMON PROPERTIES THEY
POSSESS.

5. ALPHABET ANALOGY
IN THESE TYPE OF QUESTIONS, TWO GROUPS
OF ALPHABETS RELATED TO EACH OTHER IN
THE SAME WAY ARE GIVEN.
YOU HAVE TO DECIPHER THE

RELATIONSHIP AND CHOOSE A GROUP OF


ALPHABETS THAT IS RELATED IN THE
SAME WAY TO THE THIRD GROUP IN THE
QUESTION.

ILLUSTRATIVE QUESTIONS-1-5

Directions:- (Q.Nos.1-5) Four of the following five are alike in a certain


way and hence, form a group. Which is the one that does not belong
to the group?
(1)Pear (2) Mango (3) Orange (4) Apple (5) Radish
Ans. (5) Radish
Except Radish all others are fruits. Radish is a modified root.
(2). (1)Sun (2) Moon (3) star (4) Mars (5) Universe
Ans. (5). Universe
All except universe part of the universe.
(3). (1)Tailor (2) Barber (3) Engineer (4) Carpenter (5) Black smith
Ans. (2) Barber
(4). (1)X-ray (2) Radio (3) Telephone (4)Computer (5) Television
Ans. 4. (1) X-ray
All except X-ray are media instruments.
(5). (1)Arc (2) diagonal (3) tangent (4) radius (5)Diameter
Ans. (2) Diagonal
All except diagonal are terms associated with circle

ILLUSTRATIVE QUESTIONS.6-10

Directions:- (Q.Nos.6-10) Four of the following five are alike in a certain


way and hence, form a group. Which is the one that does not belong to
the group?
Q.(6)IJK (2) ABC (3)OPQ (4) EFG (5) VWX
Ans. (5) VWX
Except VWX all other groups the first letter is the odd numbered letter in
English alphabet. I (9), A(1), O (15), E(5) V (22)
Q.(7). (1)Tiger (2) Cow (3) Bitch (4) Mare (5) Lioness
Ans. (1). Tiger
All except Tiger are of feminine gender.
Q.(8). (1)13 (2) 17 (3) 19 (4) 27 (5) 23
Ans. (5) 27 (All except 27 are prime numbers.)
Q.(9). (1)Pound (2) Yen (3) Ounce (4)Euro (5) Dollar
Ans. 3. Ounce
All except ounce are currencies of different countries.
Q.(10). (1)Lemon (2) Orange (3) Grapes (4) Banana (5) Pineapple
Ans. (4) Banana
All except Banana are juicy fruits.

ILLUSTRATIVE QUESTIONS-11-13

Directions:- (Q.Nos.11-13) The following questions consist of two words that


have certain relationship with each other, followed by 5 pairs of words. Select
the pair that has the same relationship as the original pair of words.

Q.(11). Escape : Abscond


(1). Endless : Eternal (2) Flee : Surrender (3) Exult : Jubilate (4)Escalate :
Weaken (5) Confront : Submit
Ans. (1) Endless : Eternal
Similar in meaning like that of Escape : Abscond
Q.(12). Host : Hospitable
(1)Artist : Imitative (2) Guest : Rude (3)Humanitarian : Altruistic (4)
Idealist : Cynical (5) Pollyanna : Pessimistic
Ans. (12). 3)Humanitarian : Altruistic

A s a host is supposed to be hospitable, a humanitarian is expected to be


altruistic.

Q.(13). Editor : Magazine


(1)Director : Film (2) Novel : Writer (3) Psychiatric : Neurotic (4)
Librarian : Library (5) Poem : Poet

Ans. (1) Director : Film (Like all actions of a magazine are interpreted by the
Editor, in the same way, all actions of the film are interpreted by the Director)

ILLUSTRATIVE QUESTIONS. 14-16

Directions:- (Q.Nos.14-16) In each of the following questions, the first two


words have a definite relationship. Choose one word out of the five
alternatives which will show the same relationship with the third word as
between the first two and fill the blank space.
Q.(14) As Constitution is to amendment, book is to----------(1) errata (2) contents (3)Preface (4) acknowledgement (5) Publication
Ans. (1) errata
As change in the Constitution is made by amendment, change in book is
done by errata.
Q.(15). As milk is to water, Ghee is to--------------(1)mustard oil (2) Vanaspati (3) Argemone (4) Cream (5)Hydrogenation
Ans. (2). Vanaspati
As milk is adultrated by water, in the same way, ghee is adultrated by
Vanaspati.
Q.(16). As water is to Oxygen, Salt is to.
(1)Iron (2) Sodium (3) Calcium (4)Potassium (5) Sulphur
Ans. (2)Sodium
(As Oxygen is one of the constituent of water, Sodium is a constituent of
salt.)

ILLUSTRATIVE QUESTIONS. 16A

Directions:- (Q.Nos. 16A) In each of the following questions, the first two
words have a definite relationship. Choose one word out of the five
alternatives which will show the same relationship with the third word as
between the first two and fill the blank space.
Q.(16A) As Cloud is to rain, tree is to----------(1) flower (2) stem (3) fruit (4) Root (5) leaves
Ans. (3) Fruit
As clouds produce rain, in the same way, tree produces fruit.
Q.(16B). Hospital is related to treatment, in the same way, School is related
to--------------(1) Education (2) Student (3) Teacher (4) Class room (5) Books
Ans. (1). Education
As treatment is provided in the Hospital, in the same way, Education is
provided in the school.
Q.(16). As water is to Oxygen, Salt is to.
(1)Iron (2) Sodium (3) Calcium (4)Potassium (5) Sulphur
Ans. (2)Sodium
(As Oxygen is one of the constituent of water, Sodium is a constituent of
salt.)

ILLUSTRATIVE QUESTIONS.17-18
Directions:- (Q.Nos.17-18) In each of the following questions,

there is a relations between the first two given numbers.


Another fourth figure is to be found from the given alternatives.
Choose the best alternative.

Q.(17) 6 : 18 :: 4 : ?
(1) 2 (2) 6 (3) 8 (4) 16 (5) 32
Ans. (3) 8
The relationship is :X : x2
2
Therefore ? : x2
42
2
=
2 = 16/2 = 8

Q.No.18. 9:80 :: 100: ?


(1) 901 (2) 1009 (3) 9889
Ans. (4) 9999

(4) 9999 (5) 9888

The relationship is x= (x2 1 )


Therefore, ? = (100)2 - 1 = 9999

ILLUSTRATIVE QUESTIONS.-19-21

Directions:- (Q.Nos.19-21) In each of the following questions, a group


of three inter related words is given. Choose a word from the given
alternatives that has the same relationship as the first three words.
Q.(19)Potato : Carrot :: Radish : ..
(1) Tomato (2) Spinach (3) sesame (4) Groundnut (5) Cauliflower
Ans. (4) Groundnut
All items grow under the ground.
Q.(20). Jute: cotton :: wool .
(1) Terylene (2) Silk (3) Rayon (4) Nylon (5) Polyster
Ans. (2). Silk
All are of natural fibres.
Q.(21). Sitar : Guitar :: ..
(1)Trumpet (2) Violin (3) Harmonium (4) Mridanga (5) Tabala
Ans. (2) Violin
(All are string instruments, in the same way, violin is also a string
instrument.)

ILLUSTRATIVE QUESTIONS.22-24

Directions:- (Q.Nos.22-24) The following questions consists of two


pairs of words that have certain relationship with each other, followed
by five pairs of words. Select the pair that has the same relationship
as in the original pairs of words.
Q.(22) Optimistic: Pessimistic
(1) Difficult : impossible (2) Study : Play (3) Tolerating : Disgusting
(4) Export : Import (5) Rotate : Gyrate
Ans. (4) Export : Import
The words in the question pair are opposites of each other.
Q.23). Threat : Insecurity
(1) Challenge : Fight (2) Thunder : Lightning (3) Clouds : Rain (4)
Disease : Death (5) Fire : Burn
Ans. (1) Challenge : Fight
Q.24). Tree : Branches
(1) River : Tributaries (2) Continent : Island (3) Stream : Delta (4)
Ocean : Seas (5) Chapter : Book
Ans. (1) River : Tributaries

ILLUSTRATIVE QUESTIONS.25-26

Directions:- (Q.Nos.25-26) In each of the following questions some


words are given which are related to each other in some way. The
same relationship exists among the words in one of the alternatives
given below it. Find the correct alternative.
Q.(25) Magazine: Story : Article
(1) Tea : Milk : Sugar (2) Television: Newspaper : Entertainment (3)
Bed : Quilt : Pillow (4) Novel : Drama : Literature (5) Cement : Brick :
Building
Ans. (1) Tea: Milk : Sugar
Magazine contains story and articles. Similarly, tea contains milk and
sugar.
Q.26). Village : City : Suburb
(1) Puppy : Dog : Bitch (2) School : College : University
(3) Continent : Country : State (4) Teacher : Professor : Lecturer
(5) Transistor : Radio : Television
Ans. (2) School : College : University
City and suburb are bigger and more sophisticated form of Village. In
the same way, College and Universities are bigger forms of school.

ILLUSTRATIVE QUESTIONS.27&28
Directions:- (Q.Nos.27) In each of the following question choose
a number which is similar to the numbers in the given set.
Q.(27) Given set : 282 : 354 : 444
(1) 453 (2) 417 (3) 633 (4) 255 (5) none of these
Ans. (1) 453
In all the numbers, the sum of digits is 12 and the largest digit
lies at the centre .
Directions:- (Q.Nos.28) In each of the following question choose
the set of numbers which is similar to the given set.
Q.28). Given set : (6 : 13 : 22)
(1) (6 : 13 : 27 ) (2) (10:16:28) (3) (11:18:27) (4) 13:19 :32
(5) (9:15:23)
Ans. (3) 11:18:27
In the given set second number = 1st Number + 7
3rd number = second number + 9

ILLUSTRATIVE QUESTIONS.29-32

Directions:- (Q.Nos.29) Complete the given series.


Q.(29) YEB , WFD, UHG, SKI, ?
(1) QOL (2) QGL (3) TOL (4) QNL (5) QRT
In these questions, a series of numbers or alphabetical letters are given that are
terms of the series. These terms follow a certain pattern through out the series.
You have to find the pattern and complete the series with the most suitable
alternative or find the wrong number in the series.
Sometimes, similar questions are asked on alphabetical series or letter series
also.
Ans. (1) QOL
The series is as follows.
Each element in the group is connected to the corresponding element in the next
group as follows.
i.e., Y
W
U
S
Q
E
B

F
D

H
G

Coding is a method of transmitting a message from the


sender to the receiver in a code language which cannot
be understood or comprehended by the third person.
The coded message can be deciphered or decoded by
the receiver as he knows the rule of decoding the
message.
Coding decoding test is given to judge the candidates
ability to decipher the rule which is applied for coding a
particular message and break the code to reveal the
message.
In these type of questions, some words or group of
letters and their coded forms are provided and you are
required to find the rule of coding so that you can either
encode or decode another word or another group of
letters in a similar way.

CODE BASED ON ENGLISH ALPHABET

A
Z

In order to solve questions of this type, all you needed to know is the
position of each letter from either end so that you can recognize the
coding pattern quickly.
1 2 3 4 5 6 7 8 9 10 11 12 13
A B C D E F G H I J K L M
These matrices
are useful to find
26 25 24 23 22 21 20 19 18 17 16 15 14
the coding
14 15 16 17 18 19 20 21 22 23 24 25 26
pattern of letters
N O P Q R S T U V W X Y Z
quickly.
13 12 11 10 9 8 7 6 5 4 3 2 1
Also, one must know the series of opposite alphabets

B
Y

C
X

D
W

E
V

F
U

G
T

H
S

I
R

J
Q

K
P

L
O

M
N

(1). Code based on jumbled letters of the same word.


In this type the letters of the given word are jumbled
according to certain rules to create a new coded word.
(2) Code consisting of preceding/ succeeding letter: In these type of questions, either preceding or
succeeding (following) letter stand as the code letter for
each of the letters of the given word.
Sometimes, a combination of both preceding or following
letters are used to generate the code.
Example:1. If in a certain code, LUTE is written as MUTE
and FATE is written as GATE, then how will BLUE be
written in that code?
(a) CLUE (b) GLUE (c) FLUE (d) SLUE
Ans. 1. (a) CLUE. First letter of the code word is one step
behind the corresponding letter of the word.

3.Code consisting of a group of words.


In these type of questions, a whole sentence or a group of
word is coded.
By comparing any two given sentences and its code, you
can find the code word for each word on the basis of the
common word.
Example:1. In a certain code LIM SUK TA means boys are
clever, DIN LIM PU means girls are tall and PU NE TA
means Boys and girls. which word in the language means
and
(1)LIM (2) TA (3) DIN (4) PU (5) None of these
2. If (i) A*B means A is the brother of B (ii) A0B means A is
the daughter of B (iii) A+B means A is the sister of B
Which of the following show Pis the maternal uncle
of Q ?
(1) P+R0Q (2) P*R0Q (3) QoR+P
(4) QRP (5) None of these

4. Code consisting of numbers

In these type of questions, a group of words or a whole sentence is


coded as numbers and on the basis of common words or codes, one
can find the code word for a certain word.
Eg.i. CLOCK is 42145, LEAN is 2068, CARE is 4690, then NECKLACE is
(1) 80546240 (2) 6054842 (3) 80452640 (4) 50842604 (5) 80546204
ii. If ROSE is coded as 6821, CHAIR as 73456 and PREACH is coded as
961473, what will be the code for SEARCH?
(1) 246173 (2) 214673 (3) 214763 (4) 216473 (5) 216471
R
6, O
8,S
2, E
1, C
7, H
3, A
4, I 5, P 9
Hence, SEARCH = 214673

5. Code of meaningful Words

In these type of questions, certain objects or things are assigned


different names and you are asked to find the code word for a given
word in the code language.
Eg. If sky is called sea, sea is called water, water is called air, air is called
cloud and cloud is called river, then what do we drink when thirsty?
(1) sky (2) air (3) water (4) sea (5) water

6. Code of symbols
In these type of questions, certain symbols and
signs are used for coding words or letters.
7. Code involving mathematical operations with
the position number of letters in alphabet series.
In these type of questions, the words are coded by
certain numbers.
This number is obtained by performing an
arithmetical operation of the number of each letter
in alphabet series.
Eg. 1,2,6, 24 (------)

(1) 60 (2) 120 (3) 95 (4) 150 (5) 48


The sequence is x 1 x2 x 3 x4 x5
next figure in the series is 24 x5 = 120

ILLUSTRATIVE QUESTION-1

In a certain code ABRACADABRA is coded as ZYIZXZWZYIZ.


How will the word HOCUSPOCUS be coded in that coded
language?
(1) SLXFHKLXFH (2) SLXZHKLXFH
(3) SLXFHKMXFH (4 ) SLXFHILXFH
(5) None of these
Ans. (1) SLXFHKLXFH
Obviously, the alphabet is divided into two groups. For the
first letter from the left end, the first letter from the right end is
substituted as the code and so on.
The principle is as under:A B C D E F G H I J K L M
In the same manner
H OC U S P O C U S
Z Y X WV U T S R Q P O N
SLXFHKLXFH
Therefore, H OC U S P O C U S is coded as SLXFHKLXFH

2.In a certain code language DECEMBER is written as RDEEBCME .


How will FEBRUARY be written in that code language?

(1) YFAERBRU (2) YFREABUR


(3) YFAEBRUR (4 ) YFAERUBR
(5) None of these
Ans. (2) YFREABUR
The alphabets of the given word are numbered from
the left to right and rearranged as follows: 1234 5678
81726 354
DECEMBER

RDEEBCME

1234 5678

81726 354

FE BRUARY

Y F R E A B UR

3.In a certain code language SUCCESS is written as TWFGJYZ .


How will MIRACLE be written in that code language?

(1) NKVEHRKL (2) NKUEHRL


(3) NKTEHRL (4 ) NKUHHRL
(5) None of these
Ans. (2) NKUEHRL

Note that coding is done by replacing each letter in the given word
with its succeeding letter, 2nd succeeding letter, 3rd succeeding letter
and so on. Thus ,

S U C C
E
S
S
- V DE DEF FGHI TUVWX YUVWXY
T W F G
J
Y
Z
Short cut : In such models, ascribing numbers to
the letters of the alphabet would come in handy.

1 2 3 4 5 6 7 8 9 10 11 12 13 14
A B C D E F G H I J K L M N
15 16 17 18 19 20 21 22 23 24 25 26
O P Q R S T U V W X Y Z
MIRACLE is easily coded as :M + 1 = 14 = N
I + 2 = 11 = K
R + 3 = 21 = U
A+4= 5=E
C+5= 8=H
L + 6 = 18 = R
E + 7 = 12 = L
Therefore, MIRACLE = NKUEHRL

4. In a certain code, @ implies divide, * implies subtract, $ implies


multiply and # implies addition. Then, 25$2#9$4@2*5 = ?

(1) 113
(3) 63

(2) 83
(4 ) 93

(5) None of these

Ans. (3) 63
25 $ 2 # 9 $ 4 @ 2 * 5 would be decoded as
25 x 2 + 9 x 4 2 5

= 50 + 9 x 4 5

2
= 50+18 - 5 = 63

ILLUSTRATIVE QUESTION- 5 :- In

a certain code HYD is coded as


50. How will the word NAG be coded using the same
coding system?

(1) 2
(3) 1

(2) 0.5
(4 ) 20 (5) None of these

Ans. (1) 2
Note that in HYD, on numbering, the alphabet from left
to the right with 1 to 26, H = 8, Y = 25 and D = 4.
Values of the first two letters are being multiplied and
then divided by the value of the third letter.
Thus, 8 x 25 = 50

4
In the same way, N = 14, A = 1 and G = 7.
Thus NAG = 14 x 1 = 50 =2

ILLUSTRATIVE QUESTION- 5

2. If in a certain language, MADRAS is coded as NBESBT, how is BOMBAY


coded in that language?
(a)
CPNCBX (b) CPNCBZ (c) CPOCBZ (d) CQOCBZ (e) None of these
Ans. (b) : Each letter of the word is one step behind the corresponding
letter of the code.
3. If FISH is written as EHRG in a certain code, how would JUNGLE be
written in that code?
(a) ITMFKD (b) ITNFKD (c) KVOHMF (d) TJMFKD I
Ans.3. (a : Each letter of the word is one step ahead of the corresponding
letter of the code.
4. In a certain code, TWINKLE is written as SVHOJKD, then how would
FILTE RS be written in the same code?
(a) EHKSDQR (b) EHKUDQR (c) EGKUDQR (d) GJMSFST (e) None of these
5. In a certain code, ROAD is written as URDG. How is SWAN written in that
code? (a) VXDQ (b) VZDQ (c) VZCP (d) UXDQ
6. In a certain code language, OPERATION is written as NODQBUJPO. How
is INVISIBLE written in that code?
(a) JOWJTJCMF (b) JOWJTHAKD (c) HMUHTJCL (d) HMUHTHAKD (e) None
of these

ILLUSTRATIVE QUESTION- 7-11

7, In a certain code, FAVOUR is written as EBUPTS. How is DANGER


written in that CODE?
(a) CBFFDS (b) CBMHDS (c) EBFHDS (d) EBHHFS (E) CBFNAP,
8. If SUMMER is coded as RUNNER, the code for WINTER will be .
(a)
SUITER (b) VIOUER (c) WALKER (d) SUFFER
9. In a certain code, PRODUCTIONS is written as QQPCVEUHPMT. How is
ORIENTATION written in that code?
(a) PQJDOVBSJNO (b) PQJDOUBUJPO (c) PSJFOVBSJ (d)
NSHFMVBSJNO
10. If, in a code, MIND becomes
KGLB and ARGUE becomes YPESC,
then what will be DIAGRAM be in that code?
(a) BGYEPYK (b) BGYPYEK (c) GLPEYKB (d) LKBGYPK (e) None of these
11. In a certain code, BASIC is written as DDULE. How is LEADER written
in that CODE?
(a) NGCFGT (b) NHCGGU (c) OGDFHT (d) OHDGHU

Blood Relation
Detection of relationship between persons are one
of the interesting topics in reasoning of Bank
PO/Clerk examination.
It is required to have thorough knowledge on
blood relations for solving this type of questions.
The following relationships need not be
considered.
They are step-father, stepmother, step-son, stepdaughter, step-brother, step-sister, half brother
and half sister.
Disintegration of statement, puzzles and coded
relations are the different types of problems based
on blood relations.

In these type of questions, a roundabout


complicated description is given in the form of a
certain small relationships and direct relationship
between the person concerned is to be found out.
At this stage, one need to have knowledge of
blood relations and you should try to analyze the
given statements systematically and carefully.
Eg., My mothers only child means myself.
Jayas husbands father-in-laws only daughter
means Jaya (herself).
While solving these questions, read carefully the
given information in the question.
Keep your personal biases perceived notions
aside and solve these questions by any of the two
methods hereafter forthcoming.

In solving problems of Blood Relationships, translation of


the given information in to ones own mother tongue and
substituting ones own relationships with ones owns kith
and kin is a very useful strategy.

Certain problems demand the knowledge of representing


the given statement (s) in the form of a diagram/chart.
In drawing the diagrams / charts there are no hard and fast
rules. You can use your own symbols to depict the
information, but give no room for ambiguity.
Generally, a diagram similar to the Traditional Family Tree
is used.
To be clear about gender, we use m for male and f for
female and x y z etc., for unknown.

One important aspect is that a diagram should


always clearly depict generation
Hence, horizontal lines are drawn to indicate
marriage; vertical lines indicate children, broken
lines for sibling relationships.
Take care of not to misinterpret the data, while
drawing the diagram.
A few generally used notations are :
= marriage relationship
------ = sibling relationship
| or / or \ = children
m = male, M = mother, f = female, F = Father

REMEMBER THESE IMPORTANT POINTS


AFTER READING THE QUESTION, IDENTIFY
THE TWO PERSONS BETWEEN WHOM
RELATIONSHIP IS TO BE ESTABLISHED.
TRY TO CORRELATE THE GIVEN
RELATIONSHIPS.
WHILE CONCLUDING ABOUT THE
RELATIONSHIP BETWEEN THE TWO
PERSONS; BE CAREFUL ABOUT THE
GENDERS OF THE PERSONS INVOLVED.
VARIOUS TYPES OF QUESTIONS ASKED,
DO NOT HAVE MUCH DIFFERENCE,
EXCEPT PATTERN OF QUESTIONS.

SUMMARY OF COMMON BLOOD


RELATIONSHIPS
RELATION

COMMONLY USED WORDS

Mothers or fathers brother

Uncle

Mothers or fathers sister

Aunt

Mothers or fathers father

Grandfather

Mothers or fathers mother

Grandmother

Sons wife

Daughter- in - law

Daughters husband

Son-in-law

Husbands or wifes sister

Sister-in-law

Husbands or wifes brother

Brother-in-law

RELATIONSHIPS.
RELATION

COMMONLY USED WORDS

Brothers or sisters son

Nephew
Niece

Brothers or sisters daughter


Uncles or aunts son or
daughter

Cousin

Sisters husband

Brother-in-law
Sister-in-law
Great granddaughter

Brothers wife
Grand sons or grand
daughters daughter
Grand sons or grand
daughters son

Great grandson

Sons son
Sons daughter

Grandson
Granddaughter

ILLUSTRATIVE Q UESTION- 1

If A+B means A is the mother of B; A B means A is the brother


of B; A x B means A is the son of B; and A-B means A is the
sister of B, then find the relation of the following situation.
C-PD=?

1. C is the sister of P and the mother of D


2. C is the sister of P and D
3. C is the brother of D who is the sister of D
4. C is the mother of P and D
5. None of these
Ans. 2. C is the sister of P and D
C-P = C is the sister of P
P D = P is the brother of D
Therefore, C is the sister of P and D
This can be depicted as C(f)----P(m)-----D(x)
Note that the gender of D is unknown and hence, denoted as
x.

ILLUSTRATIVE Q UESTION- 2

A man said to a woman. Your only brothers son is my


wifes brother. How is the woman related to the mans
wife?
1. sister
2. Mother-in-law
3. Aunt
4. Sister-in-law
5. None of these
Ans. 3. Aunt
The information could be deciphered as :
Woman Womans brother
|
|
Mans wife - Womans brothers son
Therefore, the woman is mans wifes fathers sister. Aunt.

ILLUSTRATIVE Q UESTION- 3
Study the following information, and answer
the questions that follow.
There are six persons P,Q,R,S,T and U in a
family. R is the sister of U. Q is the brother
of Ts husband. S is the father of P and
grand father of U. There are two fathers,
three brothers and a mother in the group.
3. Name the group of brothers.
1. Q.R.S 2. U,Q,T 3.Q,P,U 4. P,R,S 5.S.P.U
Ans. Q.P.U

The given information can be depicted as a family tree, looks like


1) R is the sister of U R(f)..U
2) Q is the brother of Ts husband Q(m).Ts husbandT(f)
3)S is the father of P and Grandfather of U. S(m) & P(m)
S(m)
P(m)

U
Also, there are two fathers, three brothers and a mother in the group. It
follows that U is male. Now, on clubbing all the above three
statements, the given data would look alike:
S(m)
Q(m)

P(m)

R(f)

U(m)

T(f)

Thus,
The Two fathers=S and P
The three brothers = Q, P and U
One mother = T

Q. 4. Who is the mother?


(1) S. (2) Q. (3) R. (4) T.
Ans. T
T is the mother of R and U

(5) None of these

Q. Who is Ts husband?
(1)P. (2) Q. (3) U. (4)S. (5) None of these
Ans. P
P is Ts husband.
Q.5. Pointing towards a person a man said to a woman.
His mother is the only daughter of your father. How
is the woman related to that person?

1. Daughter 2. Sister 3.Mother 4. Wife 5.None of


these
Ans. The only daughter of womans father is she
herself. So the person is womans son. ie., the
woman is the persons mother.
Hence, the answer is (3).

PRACTICE QUESTIONS 1-7


1. Pointing to Rajesh, Aruna said, He is the son of my fathers only son. How is
Rajeshs mother related to Aruna?
(a) Daughter (b) Aunt (c)Sister (d) Sister-in-law (e) None of these
2. Introducing Soman, Radha said, He is the only son of my mothers mother.
How is Radha related to Soman?
(a) Mother (b) Aunt (c) Sister (d) Niece (e) None of these
3. Brothers sisters husbands father-in-law is:
(a) Grandfather (b)Uncle (c) brother-in-law (d) father (e) None of these
4. Introducing Anurag to her husband, a woman said His brothers father is the
only son of my grandfather. How is the woman related to Anurag?
(a) Mother (b) Aunt (c) Sister (d) daughter (e) grandmother
5. Pointing to Reghu, Geetha said,His mothers brother is the father of my son
Balu. How is Reghu related to Geetha?
(a) Sister-in-law (b) Nephew (c) Niece (d) Aunt (e) None of these
6. Pointing to a girl on the stage, Govind said, Her mothers brother is the only son
of my mothers father. How is the girls mother related to Govind?
(a) Mother (b) Sister (c)Aunt (d) Grandmother (e) None of these
7. M introduces N saying, He is the husband of the granddaughter of the father of
my father. How is N related to M?
(a) Brother (b) Son (c) Brother-in-law (d) Nephew (e) None of these

ANSWER KEY:PRACTICE QUESTIONS 1-7


1. Pointing to Rajesh, Aruna said, He is the son of my fathers only son. How is
Rajeshs mother related to Aruna?
(a) Daughter (b) Aunt (c)Sister (d) Sister-in-law (e) None of these
2. Introducing Soman, Radha said, He is the only son of my mothers mother.
How is Radha related to Soman?
(a) Mother (b) Aunt (c) Sister (d) Niece (e) None of these
3. Brothers sisters husbands father-in-law is:
(a) Grandfather (b)Uncle (c) brother-in-law (d) father (e) None of these
4. Introducing Anurag to her husband, a woman said His brothers father is the
only son of my grandfather. How is the woman related to Anurag?
(a) Mother (b) Aunt (c) Sister (d) daughter (e) grandmother
5. Pointing to Reghu, Geetha said,His mothers brother is the father of my son
Balu. How is Reghu related to Geetha?
(a) Sister-in-law (b) Nephew (c) Niece (d) Aunt (e) None of these
6. Pointing to a girl on the stage, Govind said, Her mothers brother is the only son
of my mothers father. How is the girls mother related to Govind?
(a) Mother (b) Sister (c) Aunt (d) Grandmother (e) None of these
7. M introduces N saying, He is the husband of the granddaughter of the father of
my father. How is N related to M?
(a) Brother (b) Son (c) Brother-in-law (d) Nephew (e) None of these

Sense of Direction

Questions based on directions are one of many analytical


reasoning tests.
These questions consists of a sort of directional puzzles.
The outline describes a situation in the form of a small
passage.
You are required to arrange the scattered information
given in the outline portion of the question.
A successive follow up of directions is formulated and the
candidate has to find the final direction with respect to the
starting point or the distance between two points.
These questions are meant to judge the candidates ability
to trace, follow and sense the directions correctly.
A diagram depicting the given information help to solve
the questions quickly.

ILLUSTRATIVE Q UESTION- 1
Facing towards North Govind walks 30 metre. Then he turns
left and walks 50 metre. Then he turns left and walks 30 metre
again. Then, he turns right and walk 20 metre. How far is he
now from the starting position?
(1) 60 m (2) 35 m (3) 70 m (4) 50 m (5) None of these
Ans. 70 m

50 m

ILLUSTRATIVE Q UESTION- 2
A man is standing facing West. He turns 45 in the
clockwise direction and then another 180 in the same
direction. Then he turns 270 in the anti-clockwise
direction. Which direction is he facing now?

(1) South (2) North west (3) West


West (5) None of these

(4) SouthA

Ans. (4) South-West


The man initially faces the direction OA. O
On moving 45 clockwise, he
faces the direction OB. After
turning 180 clockwise, he
Faces direction OC. Finally
On moving 270 in the
anti-clockwise, he faces the
Direction OD which is SW.

45
180

270

ILLUSTRATIVE Q UESTION- 3

One day, Rajesh left home and cycled 15 KM southwards, turned right
and cycled 5 KM. He again turned right and cycled 15 KM and again
turned left and cycled 15 KMs. How many KMs he will have to cycle to
reach his home straight?

(1) 10km (2) 15 km (3) 20k m


these
Ans. 20 km

(4) 25km

(5) None of

ILLUSTRATIVE Q UESTION- 4

A child was looking for his father. He went 90 meters in the East before
turning to his right. He went 20 meters before turning to his right again to
look for his father at his uncles place 30 meters from this point. His father
was not there, he went 100 meters to his north before meeting his father in
the street. How far did the son meet his father from starting point?
(1) 80 m (2) 100 m (3) 140 m

Ans. (2) 100 m

(4) 260 m

(5) None of these

The child walks 30 m from A to B, then


turns right and walks 20 m up to C. Then
turns right and walks 30 m up to D. Finally
he turns right and walks 100m up to E.
AB = 90 m, BF = CD =30m
AF =AB-BF = 60 m
Also DE =100m, DF =BC = 20 m
So, EF =DE-DF =80m
Therefore, the distance from the starting
point A = AE
= AE =\/AF2+EF2 = \/(60)2+(802)
= \/3600 +6400 =\/10000 =100

ILLUSTRATIVE Q UESTION- 5
Two buses start from two opposite points of a
main road, 100 KMs apart. The first bus runs 25
kms and takes a right turn and then runs for 15
kms, it then turns left and runs for another 25 kms
and takes the direction to reach the main road. In
the meantime, due to a minor breakdown the other
bus has run only 30 kms along the main road.
What would the distance between the two buses at
this point? .
(1) 65 km (2) 20 km (3) 40 km
Ans. (2) 20 km

(4) 50km

(5) None of these

LET THE FIRST BUS BE DENOTED AS A AND THE OTHER AS B

The distance traveled by A on the main road


=25+25=50km
The distance traveled by B on the main road = 30km
The starting points are 100 KM apart.
Therefore, the buses are now at 100-(50+30) = 20km
apart.

ILLUSTRATIVE Q UESTION- 6
Directions (1-2): Study the given information carefully and answer the given
questions:
Sunil started walking from point A. He walked 6 meters towards East to reach
point B. From point B he took a right turn and walked 3 meters to reach point C.
From point C he took a right turn and walked 10 meters to reach point D. From
point D he took a left turn and walked 4 meters to reach point E. From point E, he
walked 4 meters East to reach point F. Point G is exactly mid-way between point C
and point D.

1. If Sunil walks 4 meters to the North of point F, how far and in which
direction will he be from point G?
Ans. (1) 1 meter towards East (2) 2 meters towards East (3) 1 meter
towards West (4) 2 meters towards West (5) 1.5 meters towards East

Ans. 1. (3)
Explanation:

2. How far and in which direction is


point F from point A?
Ans. (1) 7 meters towards North (2) 6 meters
towards South (3).8 meters towards South
(4) 6 meters towards North (5) None of
these
2. (5); 7 meters towards South.
Explanation:

Sense of Direction Practice Questions


(Q.1-2):Select the correct answer from the given options of direction or distance or both.

Q.1. Sreeni went 30 km to the North, then he turned West and traveled 20
KM, then he turned South and covered 10 km, then turned East and
covered 20 km. In which direction is he from the starting point?
(a) North (b) South (c) East (d) West
Ans. (a) North

2. Sambu started from his school and walks 12 km towards North. He


turns right and walks another 12 km. He turns right again and walks
12km more and turns to left to walk 6 km. How far he is from his school
and in which direction?
(a) 18km North (b) 12 Km South (c) 18 Km East (d) 12Km west

Ans. (c) 18 Km East

Sense of Direction Practice Questions


(Q.3-4):Select the correct answer from the given options of direction or distance or both.
3. Saheer Walked 20 km North. Then he turned right and walked 20km. Then he
turned right and walks 25 km. Then he turned left and walked 20 km. Finally he
turned left and walked 25km. In which direction and how many kms is he from
the starting point?
(a) 40km North (b) 60 Km East (c) 40 Km West (d) 60 Km South
Ans. (b) 60 Km East

4. Chinchu walked 5km towards East and then turned left and walked 6km. Then
she turned right and walks 10km. Finally she turned to her right and covered
6km. How far and in which direction is she from her starting point now?
(a) I5 km North (b) 15 km East (c)I5 Km South (d) I5 Km West

Ans. (b) 15 Km East

Sense of Direction Practice Questions


(Q.5-6):Select the correct answer from the given options of direction or distance or both.
5. A lady walks northwards. After a while she turns towards her right and
proceeds further towards left. Finally she turns to her left again. In which
direction is she moving now?
(a) North (b) South (c) East (d) West (e) South west
Ans. (d) West

6. John went to East up to 80 km., then he turned right and went 50 km. Then
turned right and covered 60 km. Finally he turned right and covered 50 km. How
far is he from the starting point?
(a) 50km (b) 25km (c) 20km (d) 60km (e) 40 km

Ans. (c) 20 Km

Sense of Direction Practice Questions


(Q.7-8):Select the correct answer from the given options of direction or distance or both.
Q.7. Rahim went 30 km towards East. Then he turns to his right and walked
20km. He again turns right and walked 30 km. How far and in which direction is
he from his starting point?
(a) 20km South (b) 20km North (c) 10 km South (d) 10 km North

Ans. (a) 20 KM South


Q.8. Thomas walked 4 km towards East and then turned right and walked 3 km.
Again he turned to his right and walked 3km. Again he turned to his right and
walked 3 km. Again he turned to his right and walked 4 km. After this, he turned
back 2 km. Which direction was he facing then?
(a) North (b) East (c) South (d) West

Ans. (c) South

Sense of Direction Practice Questions


(Q.9-10):Select the correct answer from the given options of direction or distance
or both.
9. Jain is 50 km south-West of Binu. If Vinay is 50 km South- East of Binu, then
Vinay is in which direction of Jain?
(a) North (b) South (c) East (d) West

Ans. (c) East


10. Sandhya walked 50 metres straight from her house which is facing North and
then walked 100 metres in the reverse direction. She then turned to her left
and covered 50 metres, In which direction is she now from her original
position?
(a) South (b) South East (c) North (d) North-West

Ans. (b) South - East

Sense of Direction Practice Questions


(Q.11-12):Select the correct answer from the given options of direction or distance
or both.
11. Seena started from her house, and covered 4km in the East direction, then
she turned to her right and Covered 3 km. What minimum distance will be
covered by her to come back to her house?
(a) 8 km (b) 4 km (c) 5 km (d) 6 km

Ans. (c) 5 KM

12. A person started from his house and walks 100 metres towards South. He
then turned left and walked 75 metres towards East. How far he is from the
starting point?
(a) 75m (b) 100m (c)125m (d)150m

Ans. (c) 125 m

Sense of Direction Practice Questions


(Q.13-14):Select the correct answer from the given options of direction or distance
or both.
13. Jameela started from her house, walked 2 km towards North, then 3 km
towards West and finally 6km towards South. The shortest distance between
her house and the ending point is...
(a) 4 km (b) 5 km (c) 6 km (d) 7 km (e) 8 KM

Ans. (b) 5 KM

14. Anagha went to the college and started from house which is in the East and
comes to a crossing. The road to her left ends in a temple straight ahead lies
the Beach. In which direction is the college if all the four are in different
directions.
(a) East (b) West (c) South (d) North

Ans. (d) North

Sense of Direction Practice Questions


(Q.15-16):Select the correct answer from the given options of direction or distance
or both.
15. Subhash is walking in the East direction. He turns 45o left and then 90o right.
In which direction is he now?
(a) North (b) East (c) North East (d) South West

Ans. (c) North East

16. James walked 3 km towards south and turned right. After walking 2 km he
turned to the left and walked 3km. In which direction was he from the starting
point?
(a) South East (b) North-East (c) South -West (d) North-West

Ans. (c) South West

Sense of Direction Practice Questions


(Q.17-18):Select the correct answer from the given options of direction or distance
or both.

17. Noushad is travelling towards North. Which of the following direction


should he take in order to travel towards south?
(a) Left, Right, Left, Left (b) Left, Left, Right, Right (c) Left, Left, Left,
Left (d) None of these

Ans. 17. (a) Left, Right, Left, Left

18. Alisha is walking towards East. What direction she should follow if
she should walk towards south?
(a) Right, Right, Left, Left, Left (b) Right, Left, Right, Left, Right (c) Right,
Right, Left, Right, Right (d) None of these

Ans. 18. (b) Right, Left, Right, Left, Right

Sense of Direction Practice Questions


(Q.19-20):Select the correct answer from the given options of direction or distance
or both.

19. Sunny is facing towards West. He turns left and covers 10 km and
turns left again and covers 10 km. Then he travels 40km towards right.
He then again turns right and covered 30km. Lastly, he turns right and
covers 50 km. How far he is from the starting point?
(a)45Km (b)35Km (c)30km (d)20km

Ans. 19. (d)2Okm


20. Ajith runs 12 km towards West, 3km towards South and then 8km
towards East. In which direction is he from the starting point?
(a) North-East (b) North-West (c) South -West (d) South East

Ans. 20. (c) South -West

IN THESE TYPES OF QUESTIONS, CLUES ARE GIVEN REGARDING


COMPARISONS AMONG A SET OF PERSONS OR THINGS WITH RESPECT TO
ONE OR MORE QUALITIES.
THE CANDIDATE HAS TO ANALYZE THE WHOLE INFORMATION AND
ESTABLISH A PROPER ASCENDING OR DESCENDING ORDER AND ANSWER
THE QUESTIONS ACCORDINGLY.
Sometimes, questions may appear very simple but they are made tricky by
infusing intricacies and complicated data.
Sometimes, you may feel that the given information is insufficient.
But a cogent and coherent analysis of it provides useful clues.
Sometimes, some superfluous statements may be provided to elude you.
Various types of questions can be asked on this.
In one of the types, you are asked find the rank of a person either from top or
bottom in a class /group.
Sometimes, the ranks of persons from either ends are given and you are asked
to find the total number of persons.
In another type questions are based on ranking; you are required to ascertain
some parameters like height, weight, marks, age etc.
To solve such types of questions, you should use some symbols and notations
to organize THE GIVEN INFORMATION so that you can use the data more
conveniently and quickly.

ILLUSTRATIVE QUESTION- 1

Vijay ranks 16th from top and 49th from bottom in a class. How many
students are there in the class?

(1) 66 (2) 65 (3) 64 (4) Cannot be determined (5) None


of these
Ans. (3) 64
In order to calculate the total number of students in the
class, we add the ranks of students from either end and
then subtract 1.
Total number of students in the class = (16+49)-1 = 64
The position of Vijay is in the class is as shown below:
Top
Bottom
15 students
48 students

ILLUSTRATIVE Q UESTION-2
In a class of 39 students, the ratio of boys to girls is 2 : 1. Reshma ranks 15 th
among all the students from the top and 8th among the girls from bottom.
How many boys are there below Reshma?
(1) 16 (2) 17 (3) 15 (4) Data inadequate (5) None of these

Ans. (2) 17
The ratio of Boys to Girls = 2:1 in a class 39 students.
Therefore, the number of boys

= 39 x 2 = 26
3

Number of Girls

= 39-26 = 13

According to the question,

14 students

15th

24students

7 Girls
Out of 24 students who are below Reshma 7 are Girls. Therefore,
number of Boys who are below Reshma = 24-7 = 17

ILLUSTRATIVE Q UESTION-3

In a row of girls, Geetha is 15th from left and Vaisali is 7th from the right.
On interchanging their places, Vaisali becomes 15th from right. How
many girls are there in the row?

(1) 27 (2) 28 (3) 29 (4) Data inadequate (5) None of these

Ans. (3) 29

Left
14 Girls

Right
15th

6 Girls

Left
14 Girls

Right

15th

6 Girls

On interchanging their places Vaisali is 15th from the right and 15th from left.
So, there are 14 girls ahead of her and 14 Girls behind her.
Therefore, total number of girls = 15+14 = 29

ILLUSTRATIVE QUESTION-4
In a class of 64 students, Sameer ranks 19 th
from the. What is his rank from the bottom?
(1) 46th (2) 45th (3) 44th (4) Data inadequate
(5) None of these
Ans. (1) 46th
According to the question,
Top
18 students
19th
46th

45 students

= (64-19)+1 = 46th
Therefore, Sameer ranks 46th from the bottom.

ILLUSTRATIVE QUESTION- 5
Reema is elder than Diana. Sunil is younger than
Reema and Rohini. Rohini is younger than Diana.
Who among them is the youngest?
(1) Reema (2) Rohini (3) Diana (4) Sunil (5)
Data inadequate
Ans. (4) Sunil
Reema > Diana..(i)
Reema, Rohini > Sunil(ii)
Diana> Rohini..(iii)
From (i), (ii) and (iii) ;
Reema > Diana > Rohini > Sunil

So, Sunil is the youngest.

Directions (6-8): Study the given information carefully and answer the
given questions:
In a group of six people, P. Q, R, S. T and U, each having a different
weight, S is heavier than Q. R is lighter than only T and P. Q is not the
lightest. P is not the heaviest.
6. Who amongst the following is the lightest?
S
(1)T (2)P (3)R (4) U (5) None of these

Ans. (4) U
7. Who amongst the following is the second heaviest?
(1) R (2)S (3)U (4) T (5)P

Ans. (5) P
8. How many people are lighter than S?
(1) None (2) Three (3) Two (4) More than three (5) One

Ans. (3) TWO

Q
T
P
R

Clues: R is
lighter than
only T and P.
Q is not the
lightest.
P is not the
heaviest.
S
Q
U

EXPLANATION

T
P
R
S
Q
U

HEAVIEST

LIGHTEST

Hence, Answers are for Q6.(4);


Q7. (5);
Q8. (3);

Directions (9). Fifteen girls are standing in a straight line facing North.
Sudha, is standing eleventh from the right end. Radha is standing ninth
from the left end. Meena is standing exactly between Sudha and Radha.
How many girls are standing to the right of Meena?

(1) Eight (2) Seven (3) Nine (4) Six (5) Cannot be determined

4 excluding Sudha (5 from L), Meena 7th, Radha 9th

11 +1=12 including Sudha

Directions (10-14): Study the given information


carefully and answer the given questions:

Eight people P, Q, R, S, T, U, V and W are sitting around a circular table facing


the centre not necessarily in the same order. T is sitting third to the right of P.
W is sitting second to the right of S. S is not an immediate neighbour of either P
or T. U and Q are immediate neighbours of each other. Q is not an immediate
neighbour of P. V is not an immediate neighbour of W.
10. What is Rs position with respect to V?
(1) Second to the left (2) Second to the right (3) Third to the right (4)
Third to the left (5) None of these
11. Four of the following five are alike in a certain way based on their
seating positions in the above arrangement and so form a group.
Which one does not belong to the group?
(l) RQ (2) PV (3) TP (4) US (5) WT
12. Who is sitting second to the right of the one who is sitting to the
immediate right of W?
(1) U (2) R (3) V (4) P (5) Q
13. How many people are sitting between U and W when counted from the
left side of U?
(1) Three (2) None (3) More than three (4) Two (5) One
14. Who is sitting exactly between T and S?
(l) U (2)Q (3)W (4)V (5)R

Explanation

10. What is Rs position with respect to V?


Ans. (2) Second to the right
11. Four of the following five are alike in a certain
way based on their seating positions in the
above arrangement and so form a group.
Which one does not belong to the group?
Ans. (l) RQ (2) PV (3) TP (4) US (5) WT
12. Who is sitting second to the right of the one
who is sitting to the immediate right of W?
Ans. (1) U (2) R (3) V (4) P (5) Q

13. How many people are sitting


between U and W when counted
from the left side of U?
Ans.(1) Three (2) None (3) More
than three (4) Two (5) One
14. Who is sitting exactly between
T and S?
Ans.(l) U (2)Q (3)W (4)V (5)R

Directions (15-19) : Study the given information carefully and


answer the given questions:

Eight people A, B, C, D, E, F, G and H are sitting in a straight line facing North


not necessarily in the same order. F is sitting fifth to the right of A. D is sitting
fifth to the right of E. E is to the immediate right of A. H is third to the left of D. B
is third to the right of G. G is an immediate neighbour of A.
15. What is Es position with respect to C?
(1) Second to the left (2) Third to the right (3) Fourth to the right (4) Third to the
left (5) None of these
16. Who is sitting third to the right of the one who is sitting sixth from the right end
of the above arrangement?
(l) D (2) C (3) E (4) H (5) None of these
17. Who amongst the following are sitting at the extreme ends of the line?
(l) G and D (2) A and D (3 ) G and F (4) E and D (5) A and B
18. Four of the following five are alike in a certain way based on their seating
positions in the above arrangement and so form a group. Which one does not
belong to the group?
(l)HD (2)BE (3)EC (4)AH (5)GB
19. If all eight persons are asked to sit in an alphabetical order from right to left,
the positions of how many will remain unchanged as compared to their original
seating position?
(1) Three (2) More than three (3) One (4) Two (5) None

Explanation

1 2 3 4 5 6 7 8

15. What is Es position with respect to C?


(1) Second to the left (2) Third to the right (3) Fourth to the right (4) Third to the left (5) None
of these
Ans. (4); Third to the left
16. Who is sitting third to the right of the one who is sitting sixth from the right end of the
above arrangement?
(l) D (2) C (3) E (4) H (5) None of these
Ans.16. (2); C
17. Who amongst the following are sitting at the extreme ends of the line?
(l) G and D (2) A and D (3 ) G and F (4) E and D (5) A and B
Ans.17.(1); (l) G and D
18. Four of the following five are alike in a certain way based on their seating positions in the
above arrangement and so form a group. Which one does not belong to the group? (l)HD
(2)BE (3)EC (4)AH (5)GB Ans.18.(2); BE
19. If all eight persons are asked to sit in an alphabetical order from right to left, the
positions of how many will remain unchanged as compared to their original seating
position? (1) Three (2) More than three (3) One (4) Two (5) None

VERBAL REASONING

These types of questions are comparatively simple


but require sharp reasoning to arrive at the
answer.

ILLUSTRATIVE QUESTION-1
1. How many pairs of letters are there in the word BUCKET which
have as many letters between them in the word as in the alphabet?

(1) One (2) Two (3) Three (4) Four


(5) More than four
Ans. (1) One
BUCKET letters in word

CDE

- Letters in the Alphabet

ILLUSTRATIVE QUESTION-2
2. How many pairs of letters are there in the word
BRIGHTEN each of which has as many letters
between them in the word as in the English
Alphabet?

(1) None (2) One (3) Two


Three (5) More than three
Ans. (3) TWO
BRIGHTEN letters in word
IJKLMN
GH

- Letters in the Alphabet

(4)

ILLUSTRATIVE QUESTION-3
Two letters in the word PRESENSE have
as many letters in them in the word as in
the alphabet and in the same order. Which
among the two letters comes earlier in the
alphabet?
(1) C (2) E (3) R (4) P (5) None of these
Ans. (4) P

PRESENSE in the word


PQRS - in the alphabet
Between P and S, P comes first

ILLUSTRATIVE QUESTION-4
From the word LAPROSCOPY, how many independent
meaningful words can be made without changing the order
of the letters and using each letter only once?(

1) One (2) Two (3) Three (4)


Four (5) More than four
Ans. (2) Two
The words that can be made from
LAPROSCOPY are LA, SCOPY or
LAP, SCOPY or LAP, COPY

ILLUSTRATIVE QUESTION-5
Which letter in the word
SELFRIGHTEOUSNESS, does not change
its position when the letters are reversed?
(1) E (2) G (3) H (4) T (5) None of these
Ans. (4) T
When the letters in the word are written in
the reverse order, the position of the middle
letter remain unchanged.
Thus in the word SELFRIGHTEOUSNESS,
the middle letter T does not change its
position when the letters are reversed.

ILLUSTRATIVE QUESTION-6
If the first and second letters in the word
DEPRESSION are interchanged, also the third
and fourth letters, the fifth and sixth letters and so
on, which of the following would be the seventh
letter from the right?
(1) R (2) O (3) S (4) I (5) None of these
Ans. (5) None of these
Given word : DEPRESSION

On interchanging the letters: EDRPSEISNO


The seventh letter from the right is P which is not
there in the answer options. Hence 5 is the
answer.

ILLUSTRATIVE QUESTION-7
If the first and fourth letters of the word SENIOR are
interchanged, also the second and fifth letters and the third and
sixth letters, how many letters are there between the letter which
is second from right and the letter which is first from the left, in the
English alphabetical order?

(1) None (2) One (3) Two (4) Three (5) More than
three
Ans. (4) Three
Given word : SENIOR
On interchanging the letters: IORSEN

The required letters are E and I [ E(FGH)I] from the


right, I and N [I(JKLM)N] and R and S which is first from
the left in the alphabetical order.

Hence (4) is the answer.

ILLUSTRATIVE QUESTION-8
Directions: (Qns. 1 - 5) The following questions are based on five
words given below.
WAX NET FOR HUB LIE

(The new words formed after performing the mentioned


operations may or may not necessarily be meaningful
English words)
1. If the positions of first and third letters of each of the words are
interchanged, which of the following will form a meaningful
English word?
(1) Both HUB and WAX (2) LIE (3) Both FOR and LIE (4) WAX (5)
NET
Ans. (5) NET
WAX XAW , NET - TEN , FOR ROF, HUB BUH, LIE EIL
Out of these, NET TEN is a meaningful word
Hence, the answer is (5)

QUESTION-82..3
If the given words are arranged in the order as they would
appear in a dictionary from left to right, which of the following
will be the fourth from the right ?

(1) FOR (2) HUB (3) LIE (4) NET (5) WAX
Ans. (2) HUB
Arranging as required : FOR, HUB, LIE, NET, WAX
Now fourth from right is HUB

Hence (2) is the answer.


Qn.3. If in each of the given words, each of the consonants changed to
previous letter and each vowel is changed to next letter in English
alphabetical series, in how many words thus formed will no vowels
appear?

(1)None (2) One (3) Two (4) Three (5) More than three
Ans. (4) Three
WAX VBW, NET MFS, FOR EPO, HUB GVA, LIE - KJF

QUESTION-845
4. If the first alphabet in each of the words is changed to next
alphabet in the English alphabetical order, how many words
having two vowels (same or different vowels) will be there ?

(1)None (2) One (3) Two (4) Three (5) four


Ans. (4) Three
Arranging as required : WAX - XAX, NET - OET, FOR GOR,
HUB IUB, LIE - MIE
Words formed having two vowels : OET, IUB, MIE

Hence (4) is the answer.


Qn.5. How many letters are there in the English alphabetical order,
between the third letter of the word which is third from the right and
third letter of the word which is second from the left of the given
words?

(1) one (2) Two (3) Three (4) four (5) five
Ans. (1) One : R,T

Classification

CLASSIFICATION MEANS TO ASSORT THE ITEMS OF A GIVEN


GROUP ON THE BASIS OF CERTAIN COMMON QUALITY THEY
POSSESS AND THEN SPOT THE ODD ONE OUT.

Letter based classifications can be in the following three


forms:
a) Based on letters b) Based on letter groups and (c)
Based on letter arrangements
In this test, you are given a group of certain items, out of
which all except one are similar to one another in some
manner.
The candidate is required to choose this one item which
does not fit into the given group.
DIFFERENT TYPES OF QUESTIONS THAT CAN BE ASKED ARE AS
FOLLOWS: (1) CHOOSE THE ODD WORD OUT
IN THESE QUESTIONS, YOU ARE GIVEN A GROUP OF CERTAIN ITEMS
OUT OF WHICH ALL EXCEPT ONE ARE SIMILAR TO ONE ANOTHER IN
SOME MANNER.
YOU HAVE TO IDENTIFY THE ITEM WHICH IS NOT FIT INTO THE GIVEN
GROUP.

ILLUSTRATIVE QUESTIONS
Directions: Choose the word which is least like the other words
in the group.
Qn.No.1. (a) Curd (b) Butter (c) Oil (d) Cheese (e) Cream
Solution:
Here all except oil are products obtained from Milk and from a
class or group.
Hence, the odd word is oil and therefore the answer is (c).
Qn.No.2. (a) Rose (b) Lotus (c) Marigold (d) Ixora (e) Lily
Solution:
Here all except Lotus are flower plants, which grow on land
and form into a class or group.
Hence, the odd word is Lotus and therefore the answer is (b).
Qn.No.3. (a) Pistol (b) Sword (c) Gun (d) Rifle (e) Cannon
Solution: Here all except Sword are fire arms and form a
class or group.
Hence, the odd word is Sword and therefore the answer is (b).

ILLUSTRATIVE QUESTIONS
Directions: In each of the following questions, five words have
been given, out of which four are alike in some manner and the
fifth one is different. Choose the word which is least like the
other words in the group.
Qn.No.4. (a) Tortoise (b) Duck (c) Snake (d) Whale (e) Crow
Solution:
Here all except Whale lay eggs and form in to a class or group.
Hence, the odd word is Whale and therefore the answer is (d).
Qn.No.5. (a) Uncle (b) Nephew (c) Brother (d) Cousin (e) Niece
Solution:
Here all except Brothers are relations based on parents,
brothers and sisters.
Hence, the odd word is Brothers and therefore the answer is (c).
Qn.No.6. (a) Reader (b) Writer (c) Publisher (d) Printer (e) Reporter
Solution: Here all except Reader are persons involved in the
preparation of a journal, news paper or magazine and form a
class or group.
Hence, the odd word is Reader and therefore the answer is (a).

ILLUSTRATIVE QUESTIONS
Directions: In each of the following questions, find out the group of
letters which is different from others.
Qn.No.7. (a) ABC (b) OPQ (c) UVW (d) HIJ (e) EFG
Solution:
Here all except HIJ the group or class is formed with a vowel at
the beginning.
Hence, the odd word is HIJ and therefore the answer is (d).
Qn.No.8. (a) R9 (b) N14 (c) M14 (d) J17 (e) Z1
Solution:
Here all except N14 are relations based on letter and its number
position from right. N14 is in neutral order.
Hence, the odd word is N14 and therefore the answer is (b).
Qn.No.9. (a) JKIJ (b) RSQR (c) BDCB (d) VWUV (e) MNLM
Solution: Here all except BDCB are in the pattern =1,-2,+1 (ie.,
J+1=K, K-2=I, I+1=J.
So, JKIJ
HENCE THE ANSWER IS (C)
Q.no.10. (a) ADE (b) FGE (c) POS (d) UVW (e) IJK
Solution: Here all except (a) are in the pattern one vowel +2
consonants So, the answer is (a) ADE

IN THESE TYPE OF QUESTIONS, YOU ARE


GIVEN CERTAIN PAIRS OF WORDS OUT OF
WHICH THE WORDS IN ALL THE PAIRS
EXCEPT ONE, BEAR A COMMON
RELATIONSHIP.
YOU HAVE TO DECIPHER THE
RELATIONSHIP AND CHOOSE THE PAIR IN
WHICH THE WORDS ARE DIFFERENTLY
RELATED THAN THE REMAINING PAIRS, AS
THE ANSWER .

ILLUSTRATIVE QUESTIONS
Directions: In each of the following questions, five pairs of words have
been given, out of which four pairs bear a certain common relationship
in some manner and the fifth pair is different. Choose the pair in which
the words are differently related.
Qn.No.1. (a) Mason : Wall (b) Cobbler : Shoe (c) Farmer : Crop (d) Chef :
Cook (e) Choreographer : Ballet
Solution:
Here all except Chef : Cook, second is prepared by the first and form in
to a class or group.
Hence, the odd pair is Chef : Cook and therefore the answer option is
(d).
Qn.No.2. (a) Blacksmith : Anvil (b) Carpenter : Saw (c) Goldsmith :
ornaments (d) Barber : Scissors (e)Sculptor : Chisel
Solution:
Here all pairs except Goldsmith : Ornaments second is the tool used
by the first.
Hence, the answer is (c).
Qn.No.3. (a) Pascal : Pressure (b) Watt : Power (c) Ampere : Current (d)
Radian : Degree (e) Joule : Energy
Solution: Here all pairs except Radian : Degree first is a unit to measure
the second and form a class or group.
Hence, the odd pair is Radian : Degree and therefore the answer is (d).

IN THESE TYPE OF QUESTIONS, YOU ARE GIVEN CERTAIN


NUMBERS OUT OF WHICH ALL EXCEPT ONE, ARE ALIKE IN
SOME MANNER WHILE ONE IS DIFFERENT.
YOU HAVE TO IDENTIFY THIS NUMBER AND THE NUMBER
IS TO BE CHOOSEN AS THE ANSWER.
Ex.1. (a) 12 (b) 25 (c) 37 (d) 49 (e) 57
Here, it is evident that 37 is the only prime number in this
group.
Hence, the answer is (c).
Ex.2. (a) 8314 (b) 2709 (c) 1315 (d) 2518 (e) 3249
Here, it is evident that in all numbers except 8314, the sum of first three
digits is equal to the units digit.
Hence, the answer is (a).
Ex.3. (a) 48 (b) 12 (c) 36 (d) 24 (e) 59
Here, it is evident that in all numbers except 59, the units digits is twice
the tens digit.

Hence, the answer is (e).

Choose the odd numeralPractice questions


Directions: In each of the following questions, five numbers
are given. Out of these, four are alike in a certain manner
but the fifth one is different. Choose the one in which is
different from the rest four.
1. (a) 43 (b) 53 (c) 63 (d) 73 (e) 83
2. (a) 10 (b) 26 (c) 24 (d) 21 (e) 18
3. (a) 51 (b) 144 (c) 64 (d) 121 (e) 256
4. (a) 15 (b) 21(c) 24 (d) 28 (e) 30
5. (a) 324 (b) 244 (c) 136 (d) 352 (e) 514
6. (a) 6 (b) 12 (c) 18 (d) 9 (e) 7
7. (a) 45 (b) 99 (c) 109 (d) 126 (e) 207
8. (a) 27 (b) 125 (c) 343 (d) 729 (e) 1321
9. (a) 21 (b) 39(c) 51 (d) 63 (e) 83
10. (a) 35 (b) 49 (c) 50 (d) 63 (e) 140
11. (a) 385 (b) 572 (c) 671 (d) 264 (e) 427
12. (a) 2384 (b) 1592 (c) 3756 (d) 4298 (e) 3629

Answer Key to Practice questions


1. (a) 43 (b) 53 (c) 63 (d) 73 (e) 83
Sol. (c) : Each of the numbers except 63, is a prime number.
2. (a) 10 (b) 26 (c) 24 (d) 21 (e) 18
Sol. (d) : Each of the numbers except 21, is an even number.
3. (a) 51 (b) 144 (c) 64 (d) 121 (e) 256
Sol. (a) : Each of the numbers except 51, is a perfect square.
4. (a) 15 (b) 21(c) 24 (d) 28 (e) 30
Sol. (d) : Each of the numbers except 28, is divisible by 3.
5. (a) 324 (b) 244 (c) 136 (d) 352 (e) 514
Sol. (a) : Sum of the digits in each other number is 10.
6. (a) 6 (b) 12 (c) 18 (d) 9 (e) 7
Sol. (e) : 7, is the only prime number in the group.
7. (a) 45 (b) 99 (c) 109 (d) 126 (e) 207
Sol. (c) : Each of the numbers except 109, is divisible by 9.
8. (a) 27 (b) 125 (c) 343 (d) 729 (e) 1321
Sol. (e) : All other numbers cubes of odd numbers.
9. (a) 21 (b) 39(c) 51 (d) 63 (e) 83
Sol. (e) : 83, is the only prime number in the group.
10. (a) 35 (b) 49 (c) 50 (d) 63 (e) 140
Sol. (c) : Each of the numbers except 50, is divisible by 7.
11. (a) 385 (b) 572 (c) 671 (d) 264 (e) 427
Sol. (e) : In all other numbers except 427, the middle digit is the sum of the other two.
12. (a) 2384 (b) 1592 (c) 3756 (d) 4298 (e) 3629
Sol. (e) : In all other numbers except 3629, the last digit is two times the first.

IN THESE TYPE OF QUESTIONS, YOU ARE GIVEN CERTAIN


PAIRS OR GROUPS OF NUMBERS OUT OF WHICH ALL
EXCEPT ONE ARE SIMILAR IN SOME MANNER WHILE ONE
IS DIFFERENT.
THE NUMBERS IN THESE SIMILAR PAIRS MAY HAVE THE
SAME PROPERTY OR MAY BE RELATED TO EACH OTHER
ACCORDING TO THE SAME RULE.
YOU HAVE TO CHOOSE THE ODD PAIR/GROUP.

Ex.1. (a) 83-75 (b) 58-50 (c) 49-42 (d) 25-17 (e) 57- 49
Here, it is evident that in each pairs except (c), the first
number is 8 more than the second.
Hence, the answer is (c).
Ex.2. (a) 70-80 (b) 54-62 (c) 28-32 (d) 21-24 (e) 14-16
Here, it is evident that in each pairs except (b), the ratio of
the two numbers is 7:8.
Hence, the answer is (b).

What is Analogy?
Analogy is a form of reasoning in which a similarity between two or more
things is inferred from a known relationship between them. As there can
be more than one relationship between two objects, it is essential to
understand which relationship the examiner wants the candidate to
locate in a particular question.
IN OTHER WORDS, ANALOGY MEANS CORRESPONDENCE.
Look at the following pair:
Ocean: Island
The pair can have the following possible relationships:
(i) The first surrounding the second. (ii) The first, a place to swim; the second
to walk. (iii) The second being very small in comparison with the first. (iv)
Specific names of ocean and island located in it.
On the basis of these relationships we can liken this pair to the following pairs
respectively
Protoplasm : Nucleus (ii). Water : Soil iii) Desert : Oasis iv) Pacific : Greenland
Or Indian : Nicobar
Thus the choice of given pairs would determine which relationship to highlight.
In case of completing the analogous pair, the word given would provide the
hint for the relationship we have to look for.

Analogy y is a verbal part of the Psychological and


Reasoning Tests and forms an important
ingredient of many competitive examinations.
VERBAL ANALOGICAL TESTS USE WORDS LETTERS AND
DIGITS .
REQUIRE COMMONSENSE REASONING TO SOLVE
QUESTIONS BASED ON THESE.
IN THESE QUESTIONS, A PARTICULAR RELATIONSHIP IS
GIVEN AND ANOTHER SIMILAR RELATIONSHIP HAS TO
BE IDENTIFIED FROM THE ALTERNATIVES PROVIDED.
THESE QUESTIONS TEST THE CANDIDATES OVERALL
KNOWLEDGE AND ABILITY TO THINK CONCISELY AND
ACCURATELY.

DIFFERENT TYPES OF QUESTIONS THAT CAN BE


ASKED ON ANALOGY ARE AS FOLLOWS
1. COMPLETING THE ANALOGOUS PAIR:
In these two words related to each other are given along
with a third word.
You have to find the relationship between the first two
words and choose the words from the given options that
bears the same relationship with the third word as the first
two words.
Two examples are given below:
1. Ship : Propeller :: Car : ?
(a) Wheel (b) Accelerator (c) Petrol (d) Steering
Explanation: (b) The second moves the first.
2. Loud : Stentorian :: Bright : ?
(a) Resplendent (b) Dull (c) Sunshine (d) Car
Explanation: (a) The second is heightened form of the first.

Directions (Qs. No. 1 to 20) : In each of the following questions the Two words
given on one side of double colon i.e., :: bear a certain relationship. The other
side shows an incomplete pair marked by a missing word which bears a similar
relationship with the given word. Find the missing word out of the given options
(a) (b) (c) or (d). Check your scores with Answers and Explanations.
1. King : Royal :: ? : Religious
(a) Prayer (b) Saint (c) Priesthood (d) Holy Boop
2. Hill : Mountain :: ? : Pain
(a) Distress (b) Discomfort (c) Headache (D) Fear
3. Raipur : Chhatisgarh:: ? : Uttaranchal
(a) Nainital (b) Ranchi (c) Dehradun (d) Bhopal
4. Eye : Cataract :: ? : Hepatitis
(a) Liver (b) Pancreas (c) Stomach (d) Lungs
5. Oxygen : Burn :: ? : Extinguish
(a) Nitrogen (b) Helium (c) Hydrogen (d) Carbon
dioxide
6. Sculptor : Statue :: ? : Poem
(a) Painter (b) Writer (c) Singer
(d) Poet
7. Jockey : Horse : : ? : Car
(a) Chauffeur (b) Tyre (c) Engine (d) Driver
8. Reading : Knowledge :: ? : Experience
(a) Job (b) Work (c) Traveler (d) Training
9. Stomata : Leaf :: ? : Bark
(a) Pore (b) Cell (c) Lenticels (d) Stem
10. Autotroph : Producer :: ? : Consumer
(a) Heterotroph (b) Carnivore (c) Herbivore (d)
Entrepreneur

11. Potato : Stem :: Radish : ?


(a) Leaf (b) Root (c) Flower (d) Seed
12. Paradigm : Pattern :: Method : ?
(a)Decoration (b) Weaving (c)System(d)print
13. Brick : Building :: ? : Letter
(a) Phrase (b) Word (c) Ink (d) Alphabet.
14. Computer : E-mail :: Mobile : ?
(a) Fax (b) SMS (c) Call (d) Ring
15. Axis : Earth :: ? : Wheel
(a) Equinox (b) Tyre (c) Hub (d) Gear
16. Reluctant : Keen :: Remarkable : ?
(a) Enthusiastic (b) Usual (c) Splendid (d)
Talented
17.Mumbai:Maharashtra::Thiruvananthapuram:?

(a) Mangalore (b) Kochi (c) Nilgiri (d) Kerala


18. Herd : Sheep :: ? : Fish
(a) Gale (b) Shoal (c) Basket. (d) Aquarium
19. Accommodation : Rent :: Travel : ?
(a) Fare (b) Ticket (c) Pass (d) Charges
20. Line : Square :: Arc : ?
(a) Semicircle (b) Circle (c) Triangle (d) Line

2. CHOOSING THE ANALOGOUS PAIR


IN THESE TYPE OF QUESTIONS, A PAIR OF WORDS IS
GIVEN, FOLLOWED BY FOUR PAIRS OF WORDS AS
OPTIONS.

YOU HAVE TO CHOOSE THE PAIR IN WHICH THE


WORDS HAVE THE SAME RELATIONSHIP TO
EACH OTHER AS THE RELATIONSHIP IN THE
GIVEN PAIR.
Ex.1. Darkness: Lamp
(a) Fatigue : Exercise (b) Thirst: Water (c) Medicine:
Illness (d) Study: Classroom (e) Leg : Socks
Sol. Just as a lamp eliminates darkness, water
eliminates thirst.
Hence, the answer is (b)

3. THREE WORD ANALOGY


IN THESE TYPE OF QUESTIONS, A GROUP
OF THREE INTER-RELATED WORDS IS
GIVEN.

YOU HAVE TO DECIPHER THE


RELATIONSHIP AMONG THESE YHREE
WORDS AND CHOOSE ANOTHER
GROUP WITH SIMILAR ANALOGY
FROM THE ALTERNATIVES THAT ARE
PROVIDED.

4. NUMBER ANALOGY
TWO TYPES OF QUESTIONS ARE ASKED ON
THIS.
1. CHOOSING A SIMILARLY RELATED PAIR
AS THE GIVEN NUMBER PAIR ON THE BASIS
OF RELATION BETWEEN THE NUMBERS IN
EACH PAIR.
2. CHOOSING A NUMBER SIMILAR TO A
GROUP OF NUMBERS ON THE BASIS OF
CERTAIN COMMON PROPERTIES THEY
POSSESS.

5. ALPHABET ANALOGY
IN THESE TYPE OF QUESTIONS, TWO GROUPS
OF ALPHABETS RELATED TO EACH OTHER IN
THE SAME WAY ARE GIVEN.
YOU HAVE TO DECIPHER THE

RELATIONSHIP AND CHOOSE A GROUP OF


ALPHABETS THAT IS RELATED IN THE
SAME WAY TO THE THIRD GROUP IN THE
QUESTION.

ILLUSTRATIVE QUESTIONS-1-5

Directions:- (Q.Nos.1-5) Four of the following five are alike in a certain


way and hence, form a group. Which is the one that does not belong
to the group?
(1)Pear (2) Mango (3) Orange (4) Apple (5) Radish
Ans. (5) Radish
Except Radish all others are fruits. Radish is a modified root.
(2). (1)Sun (2) Moon (3) star (4) Mars (5) Universe
Ans. (5). Universe
All except universe part of the universe.
(3). (1)Tailor (2) Barber (3) Engineer (4) Carpenter (5) Black smith
Ans. (2) Barber
(4). (1)X-ray (2) Radio (3) Telephone (4)Computer (5) Television
Ans. 4. (1) X-ray
All except X-ray are media instruments.
(5). (1)Arc (2) diagonal (3) tangent (4) radius (5)Diameter
Ans. (2) Diagonal
All except diagonal are terms associated with circle

ILLUSTRATIVE QUESTIONS.6-10

Directions:- (Q.Nos.6-10) Four of the following five are alike in a certain


way and hence, form a group. Which is the one that does not belong to
the group?
Q.(6)IJK (2) ABC (3)OPQ (4) EFG (5) VWX
Ans. (5) VWX
Except VWX all other groups the first letter is the odd numbered letter in
English alphabet. I (9), A(1), O (15), E(5) V (22)
Q.(7). (1)Tiger (2) Cow (3) Bitch (4) Mare (5) Lioness
Ans. (1). Tiger
All except Tiger are of feminine gender.
Q.(8). (1)13 (2) 17 (3) 19 (4) 27 (5) 23
Ans. (5) 27 (All except 27 are prime numbers.)
Q.(9). (1)Pound (2) Yen (3) Ounce (4)Euro (5) Dollar
Ans. 3. Ounce
All except ounce are currencies of different countries.
Q.(10). (1)Lemon (2) Orange (3) Grapes (4) Banana (5) Pineapple
Ans. (4) Banana
All except Banana are juicy fruits.

ILLUSTRATIVE QUESTIONS-11-13

Directions:- (Q.Nos.11-13) The following questions consist of two words that


have certain relationship with each other, followed by 5 pairs of words. Select
the pair that has the same relationship as the original pair of words.

Q.(11). Escape : Abscond


(1). Endless : Eternal (2) Flee : Surrender (3) Exult : Jubilate (4)Escalate :
Weaken (5) Confront : Submit
Ans. (1) Endless : Eternal
Similar in meaning like that of Escape : Abscond
Q.(12). Host : Hospitable
(1)Artist : Imitative (2) Guest : Rude (3)Humanitarian : Altruistic (4)
Idealist : Cynical (5) Pollyanna : Pessimistic
Ans. (12). 3)Humanitarian : Altruistic

A s a host is supposed to be hospitable, a humanitarian is expected to be


altruistic.

Q.(13). Editor : Magazine


(1)Director : Film (2) Novel : Writer (3) Psychiatric : Neurotic (4)
Librarian : Library (5) Poem : Poet

Ans. (1) Director : Film (Like all actions of a magazine are interpreted by the
Editor, in the same way, all actions of the film are interpreted by the Director)

ILLUSTRATIVE QUESTIONS. 14-16

Directions:- (Q.Nos.14-16) In each of the following questions, the first two


words have a definite relationship. Choose one word out of the five
alternatives which will show the same relationship with the third word as
between the first two and fill the blank space.
Q.(14) As Constitution is to amendment, book is to----------(1) errata (2) contents (3)Preface (4) acknowledgement (5) Publication
Ans. (1) errata
As change in the Constitution is made by amendment, change in book is
done by errata.
Q.(15). As milk is to water, Ghee is to--------------(1)mustard oil (2) Vanaspati (3) Argemone (4) Cream (5)Hydrogenation
Ans. (2). Vanaspati
As milk is adultrated by water, in the same way, ghee is adultrated by
Vanaspati.
Q.(16). As water is to Oxygen, Salt is to.
(1)Iron (2) Sodium (3) Calcium (4)Potassium (5) Sulphur
Ans. (2)Sodium
(As Oxygen is one of the constituent of water, Sodium is a constituent of
salt.)

ILLUSTRATIVE QUESTIONS. 16A

Directions:- (Q.Nos. 16A) In each of the following questions, the first two
words have a definite relationship. Choose one word out of the five
alternatives which will show the same relationship with the third word as
between the first two and fill the blank space.
Q.(16A) As Cloud is to rain, tree is to----------(1) flower (2) stem (3) fruit (4) Root (5) leaves
Ans. (3) Fruit
As clouds produce rain, in the same way, tree produces fruit.
Q.(16B). Hospital is related to treatment, in the same way, School is related
to--------------(1) Education (2) Student (3) Teacher (4) Class room (5) Books
Ans. (1). Education
As treatment is provided in the Hospital, in the same way, Education is
provided in the school.
Q.(16). As water is to Oxygen, Salt is to.
(1)Iron (2) Sodium (3) Calcium (4)Potassium (5) Sulphur
Ans. (2)Sodium
(As Oxygen is one of the constituent of water, Sodium is a constituent of
salt.)

ILLUSTRATIVE QUESTIONS.17-18
Directions:- (Q.Nos.17-18) In each of the following questions,

there is a relations between the first two given numbers.


Another fourth figure is to be found from the given alternatives.
Choose the best alternative.

Q.(17) 6 : 18 :: 4 : ?
(1) 2 (2) 6 (3) 8 (4) 16 (5) 32
Ans. (3) 8
The relationship is :X : x2
2
Therefore ? : x2
42
2
=
2 = 16/2 = 8

Q.No.18. 9:80 :: 100: ?


(1) 901 (2) 1009 (3) 9889
Ans. (4) 9999

(4) 9999 (5) 9888

The relationship is x= (x2 1 )


Therefore, ? = (100)2 - 1 = 9999

ILLUSTRATIVE QUESTIONS.-19-21

Directions:- (Q.Nos.19-21) In each of the following questions, a group


of three inter related words is given. Choose a word from the given
alternatives that has the same relationship as the first three words.
Q.(19)Potato : Carrot :: Radish : ..
(1) Tomato (2) Spinach (3) sesame (4) Groundnut (5) Cauliflower
Ans. (4) Groundnut
All items grow under the ground.
Q.(20). Jute: cotton :: wool .
(1) Terylene (2) Silk (3) Rayon (4) Nylon (5) Polyster
Ans. (2). Silk
All are of natural fibres.
Q.(21). Sitar : Guitar :: ..
(1)Trumpet (2) Violin (3) Harmonium (4) Mridanga (5) Tabala
Ans. (2) Violin
(All are string instruments, in the same way, violin is also a string
instrument.)

ILLUSTRATIVE QUESTIONS.22-24

Directions:- (Q.Nos.22-24) The following questions consists of two


pairs of words that have certain relationship with each other, followed
by five pairs of words. Select the pair that has the same relationship
as in the original pairs of words.
Q.(22) Optimistic: Pessimistic
(1) Difficult : impossible (2) Study : Play (3) Tolerating : Disgusting
(4) Export : Import (5) Rotate : Gyrate
Ans. (4) Export : Import
The words in the question pair are opposites of each other.
Q.23). Threat : Insecurity
(1) Challenge : Fight (2) Thunder : Lightning (3) Clouds : Rain (4)
Disease : Death (5) Fire : Burn
Ans. (1) Challenge : Fight
Q.24). Tree : Branches
(1) River : Tributaries (2) Continent : Island (3) Stream : Delta (4)
Ocean : Seas (5) Chapter : Book
Ans. (1) River : Tributaries

ILLUSTRATIVE QUESTIONS.25-26

Directions:- (Q.Nos.25-26) In each of the following questions some


words are given which are related to each other in some way. The
same relationship exists among the words in one of the alternatives
given below it. Find the correct alternative.
Q.(25) Magazine: Story : Article
(1) Tea : Milk : Sugar (2) Television: Newspaper : Entertainment (3)
Bed : Quilt : Pillow (4) Novel : Drama : Literature (5) Cement : Brick :
Building
Ans. (1) Tea: Milk : Sugar
Magazine contains story and articles. Similarly, tea contains milk and
sugar.
Q.26). Village : City : Suburb
(1) Puppy : Dog : Bitch (2) School : College : University
(3) Continent : Country : State (4) Teacher : Professor : Lecturer
(5) Transistor : Radio : Television
Ans. (2) School : College : University
City and suburb are bigger and more sophisticated form of Village. In
the same way, College and Universities are bigger forms of school.

ILLUSTRATIVE QUESTIONS.27&28
Directions:- (Q.Nos.27) In each of the following question choose
a number which is similar to the numbers in the given set.
Q.(27) Given set : 282 : 354 : 444
(1) 453 (2) 417 (3) 633 (4) 255 (5) none of these
Ans. (1) 453
In all the numbers, the sum of digits is 12 and the largest digit
lies at the centre .
Directions:- (Q.Nos.28) In each of the following question choose
the set of numbers which is similar to the given set.
Q.28). Given set : (6 : 13 : 22)
(1) (6 : 13 : 27 ) (2) (10:16:28) (3) (11:18:27) (4) 13:19 :32
(5) (9:15:23)
Ans. (3) 11:18:27
In the given set second number = 1st Number + 7
3rd number = second number + 9

ILLUSTRATIVE QUESTIONS.29-32

Directions:- (Q.Nos.29) Complete the given series.


Q.(29) YEB , WFD, UHG, SKI, ?
(1) QOL (2) QGL (3) TOL (4) QNL (5) QRT
In these questions, a series of numbers or alphabetical letters are given that are
terms of the series. These terms follow a certain pattern through out the series.
You have to find the pattern and complete the series with the most suitable
alternative or find the wrong number in the series.
Sometimes, similar questions are asked on alphabetical series or letter series
also.
Ans. (1) QOL
The series is as follows.
Each element in the group is connected to the corresponding element in the next
group as follows.
i.e., Y
W
U
S
Q
E
B

F
D

H
G

Das könnte Ihnen auch gefallen